Anda di halaman 1dari 63

Setelah mempelajari bab ini, siswa mampu:

1. mengonversikan satuan ukuran sudut;


2. menentukan perbandingan trigonometri pada segitiga siku-siku;
3. menentukan perbandingan trigonometri untuk sudut di berbagai kuadran;
4. menentukan perbandingan trigonometri untuk sudut-sudut berelasi;
5. menyelesaikan masalah kontekstual yang berkaitan dengan perbandingan trigonometri.
Berdasarkan pengetahuan dan keterampilan yang dikuasai, siswa mampu:
1. berpikir logis dan percaya diri dalam menyelesaikan masalah kontekstual;
2. bekerja sama dan bertanggung jawab dalam menghadapi masalah.

Satuan Ukuran
Besar Sudut
Sudut

Sudut dan Kuadran


Latihan 1 Perbandingan
Trigonometri pada
Konsep Dasar Perbandingan
Segitiga Siku-Siku
Latihan 2
Perbandingan Trigonometri
Soal-Soal Materi
Perbandingan Perbandingan Trigonometri di
Latihan 3 Berbagai Kuadran
Trigonometri
untuk Sudut-
Sudut Berelasi Perbandingan Trigonometri untuk
Ulangan Harian
Sudut-Sudut Istimewa

Perbandingan Trigonometri untuk


Sudut-Sudut Berelasi

Trigonometri
(1) Mengonversikan Satuan Derajat,
Menit, dan Detik
Tugas

Penulisan Nama Menentukan Rumusan untuk Sudut


Sudut Berelasi

Menunjukkan Hubungan Antara


Satuan Derajat dan Radian

Pemantapan Menentukan Rumus Kebalikan dan


Informasi Kegiatan
Rumus Perbandingan

Pencetus Rumusan Menentukan Nilai Sinus, Kosinus, dan


Sinus, Kosinus, dan Tangen Sudut Istimewa
Tangen
Menjelaskan Satuan Derajat dan
Radian
Berselancar
Internet Menjelaskan Sinus, Kosinus, dan
Tangen

Menentukan Koordinat Kartesius dan


Koordinat Kutub

Matematika Kelas X 1
A. Pilihan Ganda 6. Jawaban: e
Sudut selalu dihitung dari sumbu X positif yang
1. Jawaban: d diputar berlawanan arah putaran jarum jam dengan
180° titik asal (titik O) sebagai pusat putaran. Dengan
1 rad = π
sehingga:
demikian sudut 30° disajikan seperti berikut.
3 3 180°
4
π rad = 4 π× π
Y

3 45
= × 180°
41
30°
= 3 × 45° = 135° O
X
3
Jadi, sudut 4
π rad = 135°.

2. Jawaban: b
π Jadi, sudut 30° ditunjukkan oleh pilihan e.
1° = 180
rad sehingga:
7. Jawaban: e
2 π 2 Sumbu X berupa garis lurus. Garis lurus sudutnya
72° = 7 2 × rad = π rad
180°. Dengan demikian, θ + 30° = 180° ⇔ θ = 150°
5
180 5

Jadi, sudut 72° = 5


2
π rad. Jadi, besar sudut θ = 150°.
8. Jawaban: b
3. Jawaban: c
Sudut di kuadran I terletak di antara 0° dan 90°.
1 1
Sudut A = putaran = × 360° = 60° Sudut di kuadran II terletak di antara 90° dan 180°.
6 6
Sudut di kuadran III terletak di antara 180° dan
2 2 180°
Sudut B = 15
π rad = 15
π× π
= 24° 270°.
Hasil penjumlahan kedua sudut: Sudut di kuadran IV terletak di antara 270° dan
A + B = 60° + 24° = 84° 360°.
Jadi, hasil penjumlahan kedua sudut tersebut (i) Sudut α = 125° nilainya di antara 90° dan
adalah 84°. 180° sehingga terletak di kuadran II.
Pernyataan (i) benar.
4. Jawaban: d
(ii) Sudut β = 215° nilainya di antara 180° dan
11°24'32" + 89°45'56"
270° sehingga terletak di kuadran III.
= (11° + 89°) + (24' + 45') + (32" + 56")
Pernyataan (ii) salah.
= 100° + 69' + 88"
= 100° + 60' + 9' + 60" + 28" (iii) Sudut γ = –100° sama dengan sudut 260°.
= 100° + 1° + 9' + 1' + 28" Nilainya di antara 180° dan 270° sehingga
= 101°10'28" terletak di kuadran III. Pernyataan (iii) benar.
Jadi, hasil 11°24'32" + 89°45'56" = 101°10'28". (iv) Sudut θ = –30° sama dengan sudut 330°.
Nilainya di antara 270° dan 360° sehingga
5. Jawaban: b
terletak di kuadran IV. Pernyataan (iv) salah.
23°12'46" – 11°23'43" – 9°11'12"
Jadi, pernyataan yang benar adalah (i) dan (iii).
= (23° – 11° – 9°) + (12' – 23' – 11') + (46" – 43" – 12")
= (23° – 11° – 9°) + (11' – 23' – 11') + (106" – 43" – 12") 9. Jawaban: b
= (23° – 11° – 9°) + (11' – 23' – 11') + 51" Sudut A dengan 45° < A < 90°.
= (22° – 11° – 9°) + (71' – 23' – 11') + 51" (i) Sudut 2A
= (22° – 11° – 9°) + 37' + 51" 45° < A < 90°
= 2° + 37' + 51" ⇔ 2(45°) < 2(A) < 2(90°)
= 2°37'51" ⇔ 90° < 2A < 180°
Jadi, hasil 23°12'46" – 11°23'43" – 9°11'12" = Sudut 2A terletak di kuadran II.
2°37'51".

2 Trigonometri (1)
(ii) Sudut –2A 3. a. Sudut 120°
45° < A < 90° Y
⇔ –2(45°) > –2(A) > –2(90°)
⇔ –90° > –2A > –180°
⇔ –180° < –2A < –90° 120°
O X
⇔ 180° < –2A < 270°
Sudut –2A terletak di kuadran III.
Jadi, pernyataan yang benar pada pilihan b.
10. Sudut α dengan 0° < α < 90° dan sudut β dengan 3
90° < β < 180°. b. Sudut 5
π rad
(i) Pilih α = 45° dan β = 100° ternyata β ≠ 2α. 3
Pernyataan pilihan a salah. Sudut 5
π rad = 108°
(ii) Pilih α = 60° dan β = 100° diperoleh: Y
β – 2α = 100° – 2(60°) = –20° < 0°
Terdapat pasangan nilai α dan β sehingga
β – 2α kurang dari 0°. Pernyataan pilihan b 108°
X
salah. O

(iii) Pilih α = 15° dan β = 100° diperoleh:


1 1
2α + 2 β = 2(15°) + 2 (100°) = 80°
Terdapat pasangan nilai α dan β sehingga 4. a. 24°31'23" + 12°44'37"
1
2α + β kurang dari 90°. Pernyataan pilihan c = (24° + 12°) + (31' + 44') + (23" + 37")
2
= 36° + 75' + 60"
salah.
= 36° + 60' + 15' + 1'
(iv) Selisih kedua sudut: = 36° + 1° + 15' + 1'
90° – 90° < β – α < 180° – 0° = 37° + 16'
⇔ 0° < β – α < 180° = 37°16'
Pernyataan pilihan d salah. Jadi, hasil 24°31'23" + 12°44'37" = 37°16'.
(v) Jumlah kedua sudut: b. 45°15'12" – 23°14'34"
0° + 90° < α + β < 90° + 180° = (45° – 23°) + (15' – 14') + (12" – 34")
⇔ 90° < α + β < 270° = (45° – 23°) + (14' – 14') + (1' + 12" – 34")
Pernyataan pilihan e benar. = (45° – 23°) + (14' – 14') + (72" – 34")
Jadi, pernyataan yang benar pada pilihan e. = 22° + 0' + 38"
= 22°38"
B. Uraian Jadi, hasil 45°15'12" – 23°14'34" = 22°38".
1. a. Satu putaran = 360° sehingga: 5. Sudut A teletak di kuadran III sehingga
1 1 180° < A < 270°.
2
putaran = 2
× 360° = 180°
Sudut B teletak di kuadran II sehingga
180° 0° < B < 90°.
b. Satu radian = π
sehingga:
a. Hasil A + B kurang dari 360°.
15
5 5 180° Hasil jumlah kedua sudut:
π rad = π× = 75°
12 12 1 π 180° + 0° < A + B < 270° + 90°
⇔ 180° < A + B < 360°
2. a. Satu putaran = 2π rad sehingga: Jadi, pernyataan hasil A + B kurang dari 360°
1 1 1 bernilai benar.
4
putaran = 4
× 2π rad = 2
π rad
b. Hasil A – B lebih dari 90°.
π Hasil selisih kedua sudut:
b. 1° = 180
rad sehingga: 180° – 90° < A – B < 270° – 0°
7 π 7
⇔ 90° < A – B < 270°
210° = 210 × 6 rad = 6
π rad Jadi, pernyataan hasil A – B lebih dari 90°
180
bernilai benar.

Matematika Kelas X 3
c. Terdapat pasangan sudut yang memenuhi Oleh karena tidak terdapat irisan antara
A = 2B. interval sudut A dan sudut 2B maka tidak ada
Sudut A dengan 180° < A < 270°. pasangan sudut yang memenuhi.
Sudut B dengan 0° < B < 90° sehingga: Jadi, pernyataan tersebut bernilai salah.
0° < B < 90°
⇔ 2(0°) < 2(B) < 2(90°)
⇔ 0° < 2B < 180°

A. Pilihan Ganda KL 8 4
cos L = LM = 10 = 5
1. Jawaban: c KL 8 4
Segitiga PQR siku-siku di Q dengan α dan β tan M = MK = 6 = 3
masing-masing terletak pada sudut R dan 4 4
sudut P. Perbandingan trigonometri pada segitiga Jadi, nilai cos L = 5 dan tan M = .
3
tersebut sebagai berikut. 3. Jawaban: b
(i) sin α =
sisi depan sudut α
=
PQ Gambar segitiga tersebut seperti berikut.
sisi miring sudut α PR Y
sisi samping sudut α QR
(ii) cotan α = = 3
sisi depan sudut α PQ P
2 R
sisi depan sudut β QR
(iii) tan β = = PQ 1
sisi samping sudut β
sisi miring sudut β PR X
(iv) sec β = sisi samping sudut β = PQ
–4 –3 –2 –1 0 1 2 3 4
–1
sisi miring sudut β PR –2
(v) cosec β = sisi depan sudut β
= QR Q
–3
Jadi, pernyataan yang benar adalah (i), (iii), dan
(v).
Segitiga PQR siku-siku di P.
2. Jawaban: e PQ = 4 satuan
Segitiga KLM siku-siku di K seperti berikut. PR = 6 satuan
QR
Y 3 sec R 3 × PR 3QR PQ
= = ×
cosec Q QR
PR QR
M PQ
4
3PQ 3×4 12
= = = =2
PR 6 6
3 sec R
Jadi, nilai = 2.
X cosec Q
–5 0 3
4. Jawaban: e
K –2 L Nilai cos K diperoleh dari perbandingan antara
sisi KL dengan sisi KM. Pada segitiga KLM
KL = 8 satuan diperoleh panjang sisi KL, yaitu:
KM = 6 satuan
KL = KM2 − LM2
2 2
LM = KL + MK
= 82 + 62
= (2 13)2 − 42

= 64 + 36 = 52 − 16 = 36 = 6
= 100 KL 6 3 3
cos K = KM = 2 13
= 13
= 13 13
= 10 satuan
3
Jadi, nilai cos K = 13 13 .

4 Trigonometri (1)
5. Jawaban: a 7. Jawaban: c
Nilai sinus diperoleh dari perbandingan antara sisi Nilai kosinus diperoleh dari perbandingan antara
depan sudut dengan sisi miring sudut. sisi samping sudut dengan sisi miring sudut.
Berdasarkan ukuran segitiga PQR, terlihat bahwa Segitiga tersebut dapat disajikan kembali seperti
segitiga tersebut bukan siku-siku. Hal ini berikut.
ditunjukkan dengan 82 + 132 ≠ 152. Oleh karena C
itu diperlukan garis bantu, yaitu garis tinggi seperti
berikut. p
5
R
θ
A B
8 13
AB = AC2 − BC2 = p2 − 52 = p2 − 25
P Q
x O (15 – x) 2
Oleh karena cos θ = 3 , diperoleh:
Misalkan panjang PO = x cm maka panjang OQ
= (15 – x) cm. AB 2 p2 − 25 2
AC
= 3 ⇔ = 3
p
Pada segitiga POR diperoleh:
OR2 = PR2 – PO2 = 64 – x2 . . . (1) 2
⇔ 3 p − 25 = 2p
Pada segitiga OQR diperoleh:
OR2 = QR2 – OQ2 = 169 – (15 – x)2 . . . (2) ⇔ (3 p2 − 25 )2 = (2p)2
Dari persamaan (1) dan (2) diperoleh:
⇔ 9(p2 – 25) = 4p2
64 – x2 = 169 – (15 – x)2
⇔ 9p2 – 225 = 4p2
⇔ 64 – x2 = 169 – (225 – 30x + x2) ⇔ 9p2 – 4p2 = 225
⇔ 64 – x2 = 169 – 225 + 30x – x2 ⇔ 5p2 = 225
⇔ 64 – x2 = –56 + 30x – x2 ⇔ p2 = 45
⇔ 64 = –56 + 30x
⇔ p= 45
⇔ 30x = 64 + 56
⇔ 30x = 120 ⇔ p=3 5
⇔ x =4 Jadi, nilai p yang memenuhi adalah 3 5 .
Untuk x = 4 maka panjang OR:
8. Jawaban: b
OR2 = 64 – x2
Nilai kosinus diperoleh dari perbandingan antara
= 64 – 42 sisi samping sudut dengan sisi miring sudut.
= 64 – 16 = 48 Segitiga ABC dapat disajikan seperti berikut.
OR2 = 48
C
⇔ OR = 48 = 4 3
OR 4 3 4 4
sin Q = QR = = 3
13 13

4
Jadi, nilai sin Q = 3. A B
13 3

6. Jawaban: c Sisi samping sudut A adalah sisi AB dan sisi


Nilai sinus diperoleh dari perbandingan antara sisi miring sudut A adalah sisi AC. Jika panjang AB =
depan sudut dengan sisi miring sudut. Pada 3 satuan maka panjang AC = 4 satuan. Panjang
segitiga tersebut, panjang sisi depan sudut adalah BC diperoleh dengan teorema Pythagoras:
p cm dan panjang sisi miring sudut adalah 8 cm.
BC = AC2 – AB2
Dengan demikian:
3 = 42 – 32 = 16 – 9 = 7 satuan
sin β = 5
sisi samping sudut A AB 3 3
p 3 cotan A = = = = 7
⇔ 8
= 5
sisi depan sudut A BC 7 7

⇔ 5p = 24 3
Jadi, nilai cotan A = 7.
⇔ p = 4,8 7
Jadi, nilai p adalah 4,8 cm.

Matematika Kelas X 5
9. Jawaban: c 11. Jawaban: b
Nilai tangen diperoleh dari perbandingan antara 1 KL 1
sisi depan sudut dengan sisi samping sudut. Oleh cos K = a ⇔ KM = a
5 11 5 Misalkan panjang KL = 1 satuan maka panjang
karena tan P = = 11
, dapat dibentuk KM = a satuan.
11
segitiga seperti berikut. LM = a2 − 12

= a2 − 1
a a2 − 1 a2 − 1
5 Nilai sin K tan K = a
× 1
a2 − 1
=
P a
11
a2 − 1
Jadi, nilai sin K tan K= .
a
Panjang sisi miring segitiga:
12. Jawaban: b
a = 52 + ( 11)2 2 K
sin M = 3
= 25 + 11
KL 2
= 36 ⇔ KM
= 3 20

=6
20 2
sisi depan sudut P 5 ⇔ = 3 L M
sin P = sisi miring sudut P = KM
6
5 ⇔ 2KM = 3 20
Jadi, nilai sin P = 6
.
3 3×2 5
⇔ KM = 2 20 = =3 5
10. Jawaban: a 2
Nilai sinus diperoleh dari perbandingan antara sisi
Jadi, panjang sisi KM = 3 5 cm.
depan sudut dengan sisi miring sudut. Oleh
p 13. Jawaban: e
karena sin B = p = 1
, dapat dibentuk segitiga
1
seperti berikut. LΔDEF = × DF × EF
2
A 1
⇔ 9= × DF × 3
2
1 ⇔ DF = 6
p
D
DE = EF2 + DF2
C
B
= 32 + 6 2
Panjang sisi BC: = 9 + 36
BC = 2
AB – AC 2
= 45 = 3 5
EF 3 E F
= 1
12 – p2 cos E = DE = = 5
3 5 5
= 1 – p2 Jadi, cos E =
1
5.
5
sisi depan sudut B p
tan B = = 14. Jawaban: d
sisi samping sudut B 1 – p2
Perhatikan segitiga KLN.
p KN p
Jadi, nilai tan B = . sin α = LN ⇔ sin α = LN
1 – p2

p
⇔ LN = . . . (1)
sin α

6 Trigonometri (1)
12 F
Perhatikan segitiga MLN. b. D
LM LM
cos β = LN ⇔ LN = cos β . . . (2) 5

Dari persamaan (1) dan (2) diperoleh: α

p LM E
=
sin α cos β
⇔ LM sin α = p cos β DE = DF2 + EF2
p cos β
⇔ LM = sin α
= 122 + 52
p cos β = 144 + 25
Jadi, panjang sisi LM = sin α
.
= 169 = 13
15. Jawaban: c
Perhatikan ΔABC. DF 12
sin α = DE = 13
AB
AC
= cos θ EF 5
cos α = DE = 13
AB
⇔ p
= cos θ
DF 12
⇔ AB = p cos θ tan α = EF = 5
Perhatikan ΔABD. c. D
BD 24
sin θ = AB
7

BD A C
α
⇔ sin θ = p cos θ
⇔ BD = p sin θ cos θ 20
∠BAD + ∠ABD + ∠APB = 180°
⇔ θ + ∠ABD + 90° = 180° B
⇔ ∠ABD = 180° – 90° – θ
⇔ ∠ABD = (90° – θ) AC = AD2 + CD2
DE
sin ∠ABD = BD = 242 + 72
DE
⇔ sin (90° – θ) = p sin θ cos θ = 576 + 49
DE = 625
⇔ cos θ = p sin θ cos θ
⇔ DE = p sin θ cos2 θ = 25
Jadi, panjang DE = p sin θ cos2 θ.
BC = AC2 − AB2
B. Uraian
= 252 − 202
1. a. BC = AC2 − AB2
= 625 − 400
= 152 − 122 = 225

= 225 − 144 = 15
A BC 15 3
= 81 sin α = AC = 25 = 5
α
=9 AB 20 4
cos α = AC = 25 = 5
BC 9 3
sin α = AC
= 15
= 5
15 BC 15 3
12 tan α = AB = 20 = 4
AB 12 4
cos α = = =
AC 15 5 2. sin L = 0,28 M
BC 9 3 KM
tan α = AB = 12 = 4 B C ⇔ = 0,28
LM
KM 7 L
⇔ LM
= 25 K

Matematika Kelas X 7
Misalkan KM = 7 maka LM = 25, diperoleh: 4. Perhatikan segitiga KLM.
LM
KL = LM2 − KM2 tan α = KL ⇔ LM = KL tan α . . . (1)

= 252 − 72 Perhatikan segitiga NLM.


LM
= 625 − 49 tan β = NL ⇔ LM = NL tan β . . . (2)
= 576 Dari persamaan (1) dan (2) diperoleh:
KL tan α = NL tan β
= 24
⇔ (KN + NL) tan α = NL tan β
KM 7 ⇔ (1 + NL) tan α = NL tan β
a. tan L = KL = 24
⇔ tan α + NL tan α = NL tan β
7 ⇔ tan α = NL tan β – NL tan α
Jadi, tan L = 24 .
⇔ tan α = NL (tan β – tan α)
KL 24 tan α
b. tan M = KM = 7 ⇔ NL = (terbukti)
tan β − tan α
24
Jadi, tan M = 7 . 5. Bentuk persegi ABCD beserta ukurannya
disajikan seperti berikut.
3. a. sin2 C + cos2 C = 1
D C
Bukti:
c K
sin C = b a2 + c2 = b2
a
cos C = b O

c a
sin2 C + cos2 C = ( b )2 + ( b )2
c2 a2 A 12p B
= +
b2 b2
Garis AC merupakan diagonal persegi sehingga
c2 + a2
=
b2 panjang AC = 12p 2 . Titik O merupakan
b2 perpotongan kedua diagonal persegi sehingga
= =1
b2 sudut BOC merupakan sudut siku-siku. Titik O
Terbukti bahwa sin2 C + cos2 C = 1. membagi diagonal AC menjadi dua bagian sama
b. cosec2 A – cotan2 A = 1 panjang sehingga panjang OC = 6p 2 . Oleh
Bukti: karena OK : KC = 1 : 2, diperoleh:
b 1
cosec A = c 1
OK = × OC = 3 × 6p 2 = 2p 2
1+ 2
a
cotan A = c Perhatikan segitiga BOK.
b a
cosec2 A – cotan2 A = ( c )2 – ( c )2 BK = BO2 + OK 2

b2 a2 = (6p 2)2 + (2p 2)2


= –
c2 c2
b2 − a 2 = 72p2 + 8p2
=
c2
c2 = 80p2
= =1
c2
Terbukti bahwa cosec2 A – cotan2 A = 1. = 4p 5
OK 2p 2 1
sin ∠KBO = BK = = 10 10
4p 5
1
Jadi, nilai sin ∠KBO = 10 10 .

8 Trigonometri (1)
A. Pilihan Ganda 4. Jawaban: c
3π 3 × 180°
1. Jawaban: c α= 4 = 4
= 135°
Nilai perbandingan trigonometri di setiap kuadran
sebagai berikut. sin α = sin 135°
(i) Di kuadran I, sinus, kosinus, dan tangen = sin (180° – 45°)
bernilai positif. 1
= sin 45° = 2 2
(ii) Di kuadran II, hanya sinus yang bernilai
positif. cos α = cos 135°
(iii) Di kuadran III, hanya tangen yang bernilai = cos (180° – 45°)
positif. 1
(iv) Di kuadran IV, hanya kosinus yang bernilai = –cos 45° = – 2 2
positif. Oleh karena nilai sin α berlainan tanda dengan
Jadi, pernyataan yang benar adalah (i) dan (iii). cos α maka jumlah keduanya sama dengan nol.
2. Jawaban: d 1
sin α + cos α = 2 2 + (– 2 2 ) = 0.
1
Titik A(–6, 8) berada di kuadran II, dengan x = –6
dan y = 8. Jadi, pernyataan yang benar pada pilihan c.

r = OA = x2 + y2 = (−6)2 + 82 5. Jawaban: b
cos 120° + tan 315° – sin 210°
= 36 + 64 = cos (180° – 60°) + tan (360° – 45°)
– sin (180° + 30°)
= 100
= –cos 60° – tan 45° – (–sin 30°)
= 10 1 1
= – 2 – 1 – (– 2 ) = –1
Kosinus di kuadran II bernilai negatif sehingga
Jadi, nilai cos 120° + tan 315° – sin 210° = –1.
cos α = – 6 = – 3 .
10 5 6. Jawaban: e
Jadi, nilai cos α = –3 . sin 150° + sin 120° sin (180° − 30°) + sin (180° − 60°)
5
cos 210° − cos 300°
= cos (180° + 30°) − cos (360° − 60°)
3. Jawaban: a
sin 30° + sin 60°
Titik P(2, –3) berada di Y =
−cos 30° − cos 60°
kuadran IV, dengan
1 1
x = 2 dan y = –3. + 3
2 2
= 1 1
X − 3 −
r = OP = x2 + y2 O β
2 2
1 1
+ 3
2 2 2 2
= 2 + (−3) = = –1
− (1
2
3 +
1
2 )
= 4+9 P(2, –3)
sin 150° + sin 120°
Jadi, nilai cos 210° − cos 300° = –1.
= 13
Sinus di kuadran IV bernilai negatif sehingga 7. Jawaban: b
3 tan 660° – sin 900° + cos (–390)°
sin β = – =– 3 13 . = tan (360° + 300°) – sin (180° + 2 × 360°)
13 13
+ cos (360° + 30°)
Jadi, nilai sin β = – 3 13 . = tan 300° – sin 180° + cos 30°
13
= tan (360° – 60°) – 0 + cos 30°
= –tan 60° + cos 30°
1 1
=– 3 + 3 =–
2
3
2
1
Jadi, tan 660° – sin 900° + cos (–390)° = – 2 3.

Matematika Kelas X 9
8. Jawaban: c 11. Jawaban: a
BC C
Nilai kotangen diperoleh dari perbandingan antara
sin A = AC sisi samping sudut dengan sisi depan sudut. Oleh
6 3
⇔ sin 30° = AC karena cotan θ = – 4 dan θ di kuadran II, maka
6 cm
1 6
dapat dibentuk segitiga seperti berikut.
⇔ 2
= AC 30° r = (–3)2 + 4 2
A B
⇔ AC = 12
= 9 + 16
Jadi, panjang AC = 12 cm. r
4
= 25 = 5
9. Jawaban: d
Perhatikan ΔATC. Oleh karena 90° < θ < 180°
θ
maka θ di kuadran II sehingga
CT = 2 2 3
AC − AT sec θ bernilai negatif.
sisi miring sudut θ 5
= (p 3)2 − p2 sec θ = – =–3
sisi samping sudut θ
5
= 3p2 − p2 Jadi, nilai sec θ = – 3 .

= 2p2 12. Jawaban: e


C
cos (K + M) = p
= p 2 ⇔ cos (K + 90°) = p
Perhatikan ΔBCT. ⇔ –sin K = p
CT ⇔ sin K = –p
sin B = BC p 3
cos L = cos (180° – (K + M))
p 2
⇔ sin 60° = = cos (180° – (K + 90°))
BC
= cos (180° – K – 90°)
3 p 2 60°
⇔ = = cos (90° – K)
2 BC A p T B
= sin K
⇔ BC =
2p 2
×
3 2
= 3 6p = –p
3 3 sin K + cos L = –p + (–p) = –2p
2 Jadi, sin K + cos L = –2p.
Jadi, panjang BC = 3 6 p.
13. Jawaban: b
10. Jawaban: b π π
Nilai sinus diperoleh dari perbandingan antara sisi sin ( 2 + 2x) + sin ( 2 – 2x)
depan sudut dengan sisi miring sudut. Oleh π π
= sin (π – ( 2 + 2x)) + sin ( 2 – 2x)
2
karena sin α = – dan α di kuadran III, maka π π
3 = sin ( 2 – 2x) + sin ( 2 – 2x)
dapat dibentuk segitiga seperti berikut.
π
= 2 sin ( 2 – 2x)
p
p = 32 − 2 2 α = 2 cos 2x
π π
= 9−4 2
3 Jadi, sin ( 2 + 2x) + sin ( 2 – 2x) = 2 cos 2x .

= 5 14. Jawaban: b
Oleh karena α di kuadran III maka kosinus bernilai Oleh karena a + b + c = 180° dapat dibentuk:
negatif. b + c = 180° – a sehingga:
1 1
sisi samping sudut α 5 1 sin 2 (b + c) = sin 2 (180° – a)
cos α = – =– =– 5
sisi miring sudut α 3 3
1
1 = sin (90° – 2 a)
Jadi, nilai cos α = – 5.
3
1
= cos 2 a
1 1
Jadi, nilai sin 2 (b + c) = cos 2 a.

10 Trigonometri (1)
15. Jawaban: e 18. Jawaban: c
Misalkan A, B, dan C adalah sudut-sudut segitiga K = 36 C
ABC. ⇔ AB + BC + AC = 36
A + B + C = 180° ⇔ AC + AC + AC = 36
⇔ A + B = 180° – C ⇔ 3AC = 36
tan (A + B) = tan (180° – C)
⇔ AC = 12
= –tan C CD
60° 60°
1 sin A = AC A D B
cotan (A + B) =
tan (A + B) CD
1 ⇔ sin 60° = 12
=–
tan C 1 CD
= –cotan C ⇔ 2
3 =
12
Jadi, nilai cotan (A + B) = –cotan C. ⇔ CD = 6 3
16. Jawaban: c 1
LΔABC = 2 × AB × CD
p = 32 − ( 3)2 1
= 2 × 12 × 6 3
= 9−3 = 6 = 36 3
π
tan ( 2 – α) + 3 cos α Jadi, luas segitiga tersebut 36 3 cm2.
= cotan α + 3 cos α 19. Jawaban: c
3
3 x
6 6 sin 60° = 6
= + 3( )
3 3
α
1 6
= 2 + 6
p ⇔ 3 = x x
2 6
π 1
Jadi, nilai tan ( 2 – α) + 3 cos α = 2 + 6. ⇔ x=6×(2 3)
60°
17. Jawaban: d ⇔ x= 3 3
Nilai tangen diperoleh dari perbandingan antara Jadi, jarak antara ujung tangga dan permukaan
sisi depan sudut dengan sisi samping sudut. Oleh
1
tanah 3 3 m.
karena tan α = 2 , dapat dibentuk segitiga seperti
20. Jawaban: b
berikut.
C
C

α 60°
A B 30° D
2 A B
E
AC = AB2 + BC2 = 22 + 12 = 4+1 = 5 20 m

π BC
2 sin α – sin (α + 2 ) + cos (π – α) tan 30° = AB

= 2 sin α – cos α – cos α 1


⇔ BC = 3 3 (BD + 20)
= 2 sin α – 2 cos α
1 2 1 20
=2× –2× ⇔ BC = 3 3 BD + 3 3 . . . (1)
5 5
2 2 BC
=– =–5 5 tan 60° = BD
5
π BC
Jadi, nilai 2 sin α – sin (α + 2 ) + cos (π – α) ⇔ 3 = BD
2
=–5 5 . ⇔ BC = 3 BD . . . (2)

Matematika Kelas X 11
Dari persamaan (1) dan (2) diperoleh: 3 1
sin α = – =–5 5
1 20 3 5
3 BD + 3 = 3 BD
3 3
6 2
1 20
cos α = – =–5 5
3 5
⇔ 3
3 BD – 3 BD = –
3
3
3 1
2 20 tan α = 6
= 2
⇔ –3 3 BD = –
3
3
⇔ BD = 10 2. ∠BDA = 180° – (90° + 45°) = 45°
Oleh karena ∠BDA = ∠ABD = 45° maka ΔABD
Untuk BD = 10 maka BC = 3 × BD = 10 3 . siku-siku sama kaki. Akibatnya, panjang DA = AB
BE = tinggi anak = 160 cm = 1,6 m = 10 cm.
Tinggi menara = BC + BE = (10 3 + 1,6) m
DB = AB2 + DA2
Jadi, tinggi menara tersebut (10 3 + 1,6) m.
B. Uraian = 102 + 102

1. a. Titik A(4, –2) dan ∠XOA = α = 200


Y = 10 2
Perhatikan ΔCDB.
CD
cos ∠CDB = DB
α 4
X
O –2 CD
A(4, –2) ⇔ cos 60° = 10 2

1 CD
⇔ 2
= 10 2
OA = 42 + (−2)2 = 16 + 4
10 2
⇔ CD =
= 20 2

=2 5 ⇔ C D = 5 2 cm
2 1 Jadi, panjang CD = 5 2 cm.
sin α = – =–5 5
2 5
3. Jumlah sudut-sudut dalam segitiga = 180°.
4 2 ∠D + ∠E + ∠F = 180°
cos α = = 5
5
2 5 ⇔ ∠D + ∠E + 90° = 180°
1 ⇔ ∠D + ∠E = 90°
tan α = – 2 = – 2 E
4 a. Nilai sin D
b. Titik B(–3, –6) dan ∠XOB = α cos (D + F) = p
⇔ cos (D + 90°) = p
Y
⇔ –sin D = p
⇔ sin D = –p
α
Jadi, nilai sin D = –p. F D
X
O b. Nilai cos E
⇔ ∠D + ∠E = 90°
–6
⇔ ∠E = 90° – ∠D
B(–3, –6) ⇔ cos E = cos (90° – ∠D)
–3
= sin D
= –p
OB = (−3)2 + (−6)2 Jadi, cos E = –p.
= 9 + 36

= 45
=3 5

12 Trigonometri (1)
3 Perhatikan ΔAPC.
4. sin (90° – α) = 5
CP2 = AC2 – AP2
3
⇔ cos α = 5 ⇔ CP2 = 152 – 92
5 ⇔ CP2 = 225 – 81 = 144
p = 52 − 32 p
⇔ CP = 12 cm
= 25 − 9 Perhatikan ΔBPC.
α CP
= 16 3 tan B = BP
=4 12
sin (180° + α) + sin (270° + α) ⇔ tan 45° = BP
= –sin α + (–cos α) 12
4 3 7 ⇔ 1 = BP
= –5 + (– 5 ) = –5
⇔ BP = 12 cm
7
Terbukti bahwa sin (180° + α) + sin (270° + α) = – 5 . 1
Luas segitiga ABC = 2 × AB × CP
5. Perhatikan gambar berikut. 1
= 2 × (AP + BP) × CP
C
1
= 2 × (9 + 12) × 12
15 1
= 2 × 21 × 12
= 126 cm2
45°
Jadi, luas segitiga ABC = 126 cm2.
A 9 P B

Matematika Kelas X 13
Derajat
Besar Sudut

Radian

Satuan
Ukuran
Sudut Kuadran I (0°–90°)

Kuadran II (90°–180°)
Sudut dan Kuadran
Kuadran III (180°–270°)

Kuadran IV (270°–360°)

Konsep Dasar
Perbandingan

Perbandingan
Trigonometri Trigonometri
(1) Sinus
pada Segitiga
Siku-Siku Perbandingan
Trigonometri Kosinus

Tangen

Nilai Sinus Positif di Kuadran I dan II


Perbandingan
Trigonometri di
Berbagai Kuadran Nilai Kosinus Positif di Kuadran I dan IV

Perbandingan
Trigonometri Perbandingan Nilai Tangen Positif di Kuadran I dan III
untuk Sudut- Trigonometri untuk
Sudut Berelasi Sudut-Sudut Istimewa

Perbandingan
Trigonometri untuk
Sudut-Sudut Berelasi

14 Trigonometri (1)
A. Pilihan Ganda 5. Jawaban: d
12°24'11" + 21°10'7" – 29°46'12"
1. Jawaban: c = (12° + 21° – 29°) + (24' + 10' – 46') + (11" + 7" – 12")
180° = (12° + 21° – 29°) + (24' + 10' – 46') + 6"
Satu radian = π
sehingga:
= (33° – 29°) + (34' – 46') + 6"
7 7
36
180° = (32° – 29°) + (1° + 34' – 46') + 6"
π rad = π × π = (32° – 29°) + (60' + 34' – 46') + 6"
5 51
= (32° – 29°) + 48' + 6"
= 7 × 36°
= 3° + 48' + 6"
= 252°
= 3°48'6"
7 Jadi, 12°24'11" + 21°10'7" – 29°46'12" = 3°48'6".
Jadi, 5
π rad = 252°.
2. Jawaban: e 6. Jawaban: c
(i) Besar sudut α Sudut selalu dihitung dari sumbu X positif. Ada
Satu putaran = 2π rad sehingga: dua aturan yang digunakan dalam membuat
sebuah sudut.
1 1
Sudut α = 4
× 2π rad = 2
π rad (i) Untuk sudut positif diputar berlawanan arah
putaran jarum jam dengan titik asal (titik O)
(ii) Besar sudut β
sebagai pusat putaran.
π
Satu derajat = rad sehingga: (ii) Untuk sudut negatif diputar searah putaran
180
jarum jam dengan titik asal (titik O) sebagai
4 π 4
pusat putaran.
Sudut β = 240 × 3 rad = 3
π rad
180 Dengan demikian, sudut –30° disajikan pada
Hasil penjumlahan kedua sudut:
bidang koordinat seperti berikut.
1 4 11
α+β= 2
π+ 3
π= 6
π Y

Jadi, hasil penjumlahan kedua sudut tersebut


11
6
π rad. O 30° X

3. Jawaban: d
102°45'23" + 24°21'56"
= (102° + 24°) + (45' + 21') + (23" + 56") Jadi, sudut –30° ditunjukkan oleh pilihan c.
= 126° + 66' + 79"
= 126° + (60' + 6') + (60" + 19")
= 126° + (1° + 6') + (1' + 19") 7. Jawaban: d
= 127° + 7' + 19" Sudut-sudut di beberapa kuadran sebagai berikut.
= 127°7'19" 1) Sudut di kuadran I terletak di antara 0° dan
Jadi, 102°45'23" + 24°21'56" = 127°7'19". 90°.
2) Sudut di kuadran II terletak di antara 90°
4. Jawaban: a dan 180°.
34°11'24" – 17°34'45" 3) Sudut di kuadran III terletak di antara 180°
= (34° – 17°) + (11' – 34') + (24" – 45") dan 270°.
= (34° – 17°) + (10' – 34') + (1' + 24" – 45") 4) Sudut di kuadran IV terletak di antara 270°
= (34° – 17°) + (10' – 34') + (60" + 24" – 45") dan 360°.
= (34° – 17°) + (10' – 34') + 39" (i) Sudut α = 315° nilainya di antara 270° dan
= (33° – 17°) + (1° + 10' – 34') + 39" 360° sehingga terletak di kuadran IV.
= (33° – 17°) + (60' + 10' – 34') + 39" Pernyataan (i) salah.
= (33° – 17°) + 36' + 39" (ii) Sudut β = 100° nilainya di antara 90° dan
= 16° + 36' + 39" 180° sehingga terletak di kuadran II.
= 16°36'39" Pernyataan (ii) benar.
Jadi, 34°11'24" – 17°34'45" = 16°36'39".

Matematika Kelas X 15
(iii) Sudut γ = –40° sama dengan sudut 320°. Panjang sisi AC: B
Nilainya di antara 270° dan 360° sehingga
terletak di kuadran IV. Pernyataan (iii) salah. AC = AB2 – BC2
(iv) Sudut θ = –120° sama dengan sudut 240°. 6
5
Nilainya di antara 180° dan 270° sehingga = 62 – 52
terletak di kuadran III. Pernyataan (iv) benar. = 36 – 25 = 11 A C
Jadi, pernyataan yang benar adalah (ii) dan (iv).
sisi depan sudut B
8. Jawaban: a tan B = sisi samping sudut B
Segitiga ABC disajikan pada bidang koordinat
seperti berikut. AC
= BC
Y
11
3 =
C 5
2
1 1
= 11
X 5
–4 –3 –2 –1 0 1 2 3 4
–1
1
A –2 B Jadi, nilai tan B = 11 .
5

Segitiga ABC siku-siku di B. 11. Jawaban: c


AB = 6 satuan Nilai sinus diperoleh dari perbandingan antara
BC = 4 satuan sisi depan sudut dengan sisi miring sudut. Oleh
AC = 62 + 4 2 karena sin 15° = p maka dapat dibentuk segitiga
seperti berikut.
= 36 + 16
1
p
= 52 15°
= 2 13 x

AB 6 3 Jika sisi depan sudut adalah p satuan, sisi miring


cos A = AC = 2 13
= 13 13 sudut adalah 1 satuan.
3 Panjang sisi samping sudut:
Jadi, nilai cos A = 13 13 .
x= 12 – p 2 = 1 – p2
9. Jawaban: c
Nilai sinus α diperoleh dari perbandingan antara tan 15° =
sisi depan sudut 15°
=
p
sisi depan sudut α dengan sisi miring sudut α. sisi samping sudut 15° 1 – p2
Panjang sisi miring:
p
Jadi, nilai tan 15° = .
AC = AB2 + BC2 1 – p2

= 62 + 92 12. Jawaban: d
1 C
= 36 + 81 tan α = 2
BC 1 2 5
= 117 ⇔ AB
= 2 x
= 3 13 ⇔
x
=
1
α
AB 2 A B
BC 9 3
sin α = = = 13 ⇔ AB = 2x
AC 3 13 13
AB2 + BC2 = AC2 (tripel Pythagoras)
3
Jadi, nilai sin α = 13 . ⇔ (2x)2 + x2 = (2 5 )2
13
⇔ 4x2 + x2 = 20
10. Jawaban: a
Nilai sinus diperoleh dari perbandingan antara ⇔ 5x 2 = 20
sisi depan sudut dengan sisi miring sudut. Oleh ⇔ x2 = 4
5 ⇔ x=2
karena sin A = 6 maka dapat dibentuk segitiga
Jadi, nilai x = 2.
ABC seperti berikut.

16 Trigonometri (1)
13. Jawaban: b 15. Jawaban: b
Nilai sinus diperoleh dari perbandingan antara Misalkan panjang BC = CD = p.
sisi depan sudut dengan sisi miring sudut. Oleh Perhatikan segitiga ACD siku-siku di D.
karena sin θ = 0,8 diperoleh: AD
12
tan α = CD
⇔ AD = CD tan α = p tan α
sin θ = 0,8 ⇔ r
= 0,8
Pada segitiga ABD siku-siku di D berlaku

12
=
8 teorema Pythagoras, diperoleh:
r 10
⇔ 8r = 120 AB = AD2 + BD2
⇔ r = 15
Nilai p: = (p tan α)2 + (2p)2

p= 152 – 122 = 225 – 144 = 81 = 9 = p 2 tan2 α + 4p 2


Jadi, nilai p + r = 9 + 15 = 24.
= p 2 (4 + tan2 α)
14. Jawaban: b
Cara I: 2
= p 4 + tan α
Nilai tangen diperoleh dari perbandingan antara
sisi depan sudut dengan sisi samping sudut. Oleh sisi depan sudut B AD
2
sin B = sisi miring sudut B = AB
karena tan β = 3
maka dapat dibentuk segitiga
seperti berikut. p tan α
=
p 4 + tan2 α
r
2 tan α
=
4 + tan2 α
β
3 tan α
Jadi, sin B = .
4 + tan2 α
r = 2 + 3 = 4 + 9 = 13
2 2

Dengan demikian diperoleh: 16. Jawaban: a


sisi depan sudut β 2 Titik A di kuadran III.
sin β = = Panjang OA:
sisi miring sudut β 13

cos β =
sisi samping sudut β
=
3 OA = (–3)2 + (–4)2
sisi miring sudut β 13
2 3 = 9 + 16
5 sin β – 6 cos β 5( )–6( )
13 13
Nilai = 3 2 = 25
2 cos β + 3 sin β 2( )+3( )
13 13
=5
10

18 Cermati gambar berikut.
13 13
= 6 6
+ Y
13 13

8 2
= – 12 = – 3
Cara II: N 3
X
1 β O
5 sin β – 6 cos β 5 sin β – 6 cos β cos β 4
= × 1 5
2 cos β + 3 sin β 2 cos β + 3 sin β
cos β
sin β
5 cos β – 6 cos β
cos β A
= cos β sin β
2 cos β + 3 cos β
Misalkan sudut AON = β diperoleh:
5 tan β – 6
= 2 + 3 tan β
AN 4
2 10
sin β = OA
= 5
5 (3) – 6 –6
= 2 = 3
2 + 3 (3) 2 +2 4
10 18
– 3
sin α = sin (180° + β) = –sin β = – 5
8 2
= 3
= – 12 = – 3
4 4
5 sin β – 6 cos β 2 Jadi, nilai sin α = – 5 .
Jadi, nilai = –3 .
2 cos β + 3 sin β

Matematika Kelas X 17
17. Jawaban: a 21. Jawaban: b
5 Nilai tangen diperoleh dari perbandingan antara
Sudut 6
π = 150°. sisi depan sudut dengan sisi samping sudut. Nilai
12
cos 150° = cos (180° – 30°) tan θ = dapat disajikan dalam segitiga seperti
5
= –cos 30° berikut.
1
=–2 3
5 1
Jadi, nilai cos 6
π = –2 3 . r
12
18. Jawaban: b
θ
tan (–1.200°) 5
= –tan 1.200°
= –tan (3 × 360° + 120°) Nilai r:
= –tan 120°
= –tan (180° – 60°) r = 122 + 52 = 144 + 25 = 169 = 13
= tan 60° tan (270° – θ) + cos (180° + θ)
= = cotan θ – cos θ
3
5 5
Jadi, nilai tan (–1.200°) = = – 13
3. 12
5
19. Jawaban: b = 156
π 5π 3π π 5
cos2 – sin2 + 8 cos sin Jadi, nilai tan (270° – θ) + cos (180° + θ) = 156
.
6 6 4 4
= cos2 30° – sin2 150° + 8 cos 135° sin 45° 22. Jawaban: c
= cos2 30° – sin2 (180° – 30°) Nilai kosinus diperoleh dari perbandingan antara
+ 8 cos (180° – 45°) sin 45° sisi samping sudut dengan sisi miring sudut. Nilai
= cos2 30° – sin2 30° + 8 (–cos 45°) sin 45° 5
cos α = 5
maka dapat disusun segitiga seperti
1 1 2 1 1
=( 3 )2 – ( ) + 8(– 2 )( 2)
2 2 2 2 berikut.
3 1
= 4
– 4
–4
5
1 p
= –3 2
π 5π 3π π α
Jadi, nilai cos2 6
– sin2 6
+ 8 cos 4
sin 4
=
5
1
–3 2 .
Nilai p:
20. Jawaban: e
sin 150° + cos 330° p= 52 − ( 5)2
tan 225° − sin 300°
= 25 − 5
sin (180° − 30°) + cos (360° − 30°)
= tan (180° + 45°) − sin (360° − 60°) = 20
sin 30° + cos 30°
= tan 45° − ( − sin 60°) = 2 5
π p 2 5
1
+
1
3 cotan ( 2 – α) = tan α = 5
= =2
= 2 2
1
5
1+ 3 π
2 Jadi, nilai cotan (2 – α) = 2.
1+ 3 2− 3
= 2+ 3
× 2− 3 23. Jawaban: a
Jumlah sudut segitiga = 180°
2− 3 +2 3 −3
= ⇔ A + B + C = 180°
4−3
⇔ A + B = 180° – C
= 3 –1
sin (A + B)= sin (180° – C) = sin C
sin 150° + cos 330° Jadi, nilai sin (A + B) = sin C.
Jadi, nilai tan 225° − sin 300°
= 3 – 1.

18 Trigonometri (1)
24. Jawaban: c 28. Jawaban: c
α + β = 270° ⇔ α = 270° – β Segitiga KLN siku-siku di L.
cos α + sin β NL
= cos (270° – β) + sin β sin K = KN
= –sin β + sin β
NL
=0 ⇔ sin 45° = 6
Jadi, nilai cos α + sin β = 0.
1 NL
25. Jawaban: c ⇔ 2 =
2 6
sin 25°
tan 25° × tan 65° – cos 65° ⇔ NL = 3 2 cm
sin 25° Segitiga NLM siku-siku di L.
= tan 25° × tan (90 – 25)° – cos (90 − 65)°
NL
sin 25° tan L = LM
= tan 25° × cotan 25° – sin 25°
sin 25° 3 2
= tan 25° ×
1
– sin 25° ⇔ tan 60° =
tan 25° LM

=1–1=0 3 2
⇔ 3 =
sin 25° LM
Jadi, nilai tan 25° × tan 65° – cos 65°
= 0.
3 2
⇔ LM = = 6 cm
26. Jawaban: a 3

cos 110° × cotan 160° + sin 200° Jadi, panjang LM = 6 cm.


= cos (180 – 70)° × cotan (180 – 20)° + sin (180 + 20)° A
29. Jawaban: c
= –cos 70° × (–cotan 20°) – sin 20° Oleh karena AB = AC,
= –cos 70° × (–cotan (90 – 70)°) – sin (90 – 70)° segitiga ABC sama kaki.
= –cos 70° × (–tan 70°) – cos 70° ∠B = ∠C = 75°
sin 70° ∠A + ∠B + ∠C = 180°
= –cos 70° × (– cos 70° ) – cos 70°
⇔ ∠A + 75° + 75° = 180°
= sin 70° – cos 70°
⇔ ∠A + 150° = 180°
=p–q 75°

Jadi, nilai cos 110° × cotan 160° + sin 200° = p – q. ⇔ ∠A = 30° B C

27. Jawaban: c Jadi, nilai cos A = cos 30° = 1 3 .


2
Untuk menentukan panjang PS digunakan
30. Jawaban: b
teorema Pythagoras pada segitiga PSR. Oleh
LM
karena hanya diketahui panjang PR maka ukuran cos M = KM M
RS dicari terlebih dahulu. Perhatikan segitiga
LM
QSR. ⇔ cos 60° = 12 cm
12 60°
RS RS
sin Q = ⇔ sin 30° = 1 LM
QR 40 ⇔ = 12
2 K L
1 RS
⇔ 2
= ⇔ L M = 6 cm
40
Jadi, panjang LM = 6 cm.
⇔ RS = 20
B. Uraian
Diperoleh panjang RS = 20 cm.
1
PS = PR2 − RS2 1. a. putaran
3
Satu putaran = 360° sehingga:
= 252 − 20 2
1 1
putaran = × 360° = 120°
= 625 − 400 3 3

2
= 225 b. 5
putaran
= 15
2 2
Jadi, panjang PS = 15 cm. putaran = × 360° = 144°
5 5

Matematika Kelas X 19
2 3. a. 13°34'12" + 34°45'54"
c. 5
π rad = (13° + 34°) + (34' + 45') + (12" + 54")
180°
= 47° + 79' + 66"
1 rad = π
sehingga: = 47° + (60' + 19') + (60" + 6")
= 47° + (1° + 19') + (1' + 6")
2 2 180°
π rad = π× = 72° = 48° + 20' + 6"
5 5 π
= 48°20'6"
180° Jadi, 13°34'12" + 34°45'54" = 48°20'6".
d. 1 rad = π
sehingga
b. 102°16'9" – 34°26'45" – 19°12'51"
7 7 180° = (102° – 34° – 19°) + (16' – 26' – 12')
4
π rad = 4
π× π
= 315°
+ (9" – 45" – 51")
2. Sudut selalu dihitung dari sumbu X positif. Sudut = 49° + (16' – 26' – 12') + (9" – 45" – 51")
positif diputar berlawanan arah putaran jarum jam = 49° + (14' – 26' – 12') + (2' + 9" – 45" – 51")
sedangkan sudut negatif diputar searah putaran = 49° + (14' – 26' – 12') + (120" + 9" – 45" – 51")
jarum jam. = 49° + (14' – 26' – 12') + 33"
a. Sudut 105° = 48° + (1° + 14' – 26' – 12') + 33"
Arah sudut berlawanan arah putaran jarum = 48° + (60' + 14' – 26' – 12') + 33"
jam. = 48° + 36' + 33"
Y = 48°36'33"
Jadi, 102°16'9" – 34°26'45" – 19°12'51" =
48°36'33".
105° 4. a. Y
X
O
5
4
C(4, 3)
3
2
b. Sudut –35° 1
X
Arah sudut searah putaran jarum jam. –5 –4 –3 –2 –1 0 1 2 3 4 5
–1
Y –2
A(–4, –3) –3
B(4, –3)
–4
–5

35° X
O
b. AB = 4 – (–4) = 8 satuan
BC = 3 – (–3) = 6 satuan

AC = AB2 + BC2
4
c. Sudut 3
π rad
= 82 + 62
4 4 180°
Sudut π rad = π× = 240° 64 + 36
3 3 π =
Arah sudut berlawanan arah putaran jarum
jam. = 100 = 10 satuan
BC 6 3
Y sin A = AC = 10 = 5
AB 8 4
cos A = AC = 10 = 5
240° BC 6 3
X tan A = AB = 8 = 4
O
3 4
Jadi, nilai sin A = 5 , cos A = 5 , dan
3
tan A = 4 .

20 Trigonometri (1)
5. Segitiga ABD siku-siku di B dengan AB = 5 cm 7. Cara I:
dan AD = 13 cm. Panjang BD dicari dengan Segitiga ABD siku-siku di B dengan ∠BDA = 30°
teorema Pythagoras: dan panjnag AD = 24 cm.
(i) Panjang AB
BD = AD2 – AB2
AB
2 2
sin ∠BDA = AD
= 13 – 5
AB
= 169 – 25 ⇔ sin 30° = 24

= 144 1 AB
⇔ 2
= 24
= 12 cm
⇔ AB = 12 cm
Segitiga BCD siku-siku di D dengan BC = 20 cm
(ii) Panjang BD
dan BD = 12 cm. Panjang CD dicari dengan
teorema Pythagoras: BD
cos ∠BDA = AD
CD = BC2 – BD2 BD
⇔ cos 30° = 24
= 2
20 – 12 2
1 BD
⇔ 3 =
= 400 – 144 2 24

= 256 ⇔ BD = 12 3 cm
= 16 cm (iii) Panjang BC
(i) sin α Segitiga ABC siku-siku di B dengan ∠CAB
sisi depan sudut α CD 16 4 = 30° dan panjang AB = 12 cm.
sin α = = BC
= 20
= 5
sisi miring sudut α BC
(ii) cos α tan ∠CAB = AB
sisi samping sudut α BD 12 3
cos α = sisi miring sudut α = BC
= 20
= 5 BC
⇔ tan 30° =
(iii) tan α 12
sisi depan sudut α CD 16 4 1 BC
tan α = = BD = 12 = 3 ⇔ 3 =
sisi samping sudut α 3 12
4 3 4
Jadi, nilai sin α = 5 , cos α = 5 , dan tan α = 3
. ⇔ BC = 4 3 cm
Panjang CD:
6. Hitunglah nilainya.
a. tan 315° – cos 240° + sin 150° CD = BD – BC = 12 3 – 4 3 = 8 3 cm
sin 120° × cos 300° Cara II:
b. tan 210° Jumlah sudut dalam segitiga ABC adalah 180°,
Jawaban: diperoleh:
a. tan 315° – cos 240° + sin 150° ∠BAC + ∠ABC + ∠BCA = 180°
= tan (360° – 45°) – cos (180° + 60°) ⇔ 30° + 90° + ∠BCA = 180°
+ sin (180° – 30°) ⇔ 120° + ∠BCA = 180°
= –tan 45° – (–cos 60°) + sin 30° ⇔ ∠BCA = 60°
1 1 ∠BCA dan ∠ACD saling berpelurus sehingga:
= –1 + 2 + 2
∠BCA + ∠ACD = 180°
=0 ⇔ 60° + ∠ACD = 180°
Jadi, tan 315° – cos 240° + sin 150° = 0. ⇔ ∠ACD = 120°
sin 120° × cos 300°
b. Jumlah sudut dalam segitiga ACD adalah 180°,
tan 210°
diperoleh:
sin (180° − 60°) × cos (360° − 60°)
= ∠ACD + ∠CDA + ∠DAC = 180°
tan (180° + 30°)
⇔ 120° + 30° + ∠DAC = 180°
sin 60° × cos 60°
1
2

1
2 3 ⇔ 150° + ∠DAC = 180°
= tan 30° = 1 = ⇔ ∠DAC = 30°
3
3 4
Oleh karena ∠DAC = ∠CDA = 30° maka segitiga
sin 120° × cos 300° 3 ACD sama kaki sehingga AC = CD.
Jadi, tan 210° = .
4

Matematika Kelas X 21
Dari langkah pada cara I diperoleh panjang AB = Nilai a:
12 cm.
a= (p 2 + 1) – 1 = p2 = p
AB
cos ∠BAC = AC tan 95° + tan 185°
tan 275° + tan 355°
12
⇔ cos 30° = AC tan (90° + 5°) + tan (180° + 5°)
= tan (270° + 5°) + tan (360° – 5°)
1 12
⇔ 3 = –cotan 5° + tan 5°
2 AC
= –cotan 5° – tan 5°
⇔ 3 AC = 24 –
1 p
+
p 1
=
⇔ AC = 8 3 cm 1 p
– –
p 1
–1 + p2
Jadi, panjang CD = AC = 8 3 cm. p
= –1 – p2
8. a. Panjang QS p
Sisi QS merupakan sisi miring segitiga siku-
–1 + p2
siku PQS. = –1 – p 2
PQ 20
cos ∠PQS = QS
⇔ cos 45° = QS
tan (90° + 5°) + tan (180° + 5°) –1 + p2
Jadi, tan (270° + 5°) + tan (360° – 5°)
= –1 – p 2
.
2 20
⇔ = QS
2
40 π
⇔ QS = 10. Oleh karena 0 < θ < 2 maka θ terletak di kuadran
2
⇔ QS = 20 2 I. Nilai tangen diperoleh dari perbandingan antara
sisi depan sudut dengan sisi samping sudut. Nilai
Jadi, panjang QS = 20 2 cm. 1
tan θ = 5 dapat dibentuk segitiga berikut.
b. Panjang QR 2
Sisi QR merupakan sisi segitiga siku-siku
QRS. r
5
QR QR
sin ∠SQR = QS
⇔ sin 60° = 20 2 θ
3 QR 2
⇔ = 20 2 Nilai r:
2
20 2 × 3
⇔ QR = r= ( 5)2 + 22 = 5+4 = 9 =3
2
⇔ QR = 10 6 sec (

– θ) + cotan (2π – θ)
Nilai 2
π
Jadi, panjang QR = 10 6 cm. cos ( 2 + θ) – tan (π – θ)

9. Nilai kosinus diperoleh dari perbandingan antara sec (270° – θ) + cotan (360° – θ)
sisi samping sudut dengan sisi miring sudut. Nilai = cos (90° + θ) – tan (180° – θ)
cos 5° = sehingga dapat dibentuk segitiga seperti –cosec θ – cotan θ
berikut. = –sin θ + tan θ
3 2 5
– – –
p2 + 1 = 5 5
= 5
= –6
a –
5
+
5 5
3 2 6



1 sec ( – θ) + cotan (2π – θ)
Jadi, nilai 2
π
= –6.
cos ( 2 + θ) – tan (π – θ)

22 Trigonometri (1)
Setelah mempelajari bab ini, siswa mampu:
1. menjelaskan identitas dasar trigonometri sebagai hubungan antara rasio trigonometri dan perannya dalam membuktikan identitas
trigonometri lainnya;
2. menggunakan identitas dasar trigonometri untuk membuktikan identitas trigonometri lainnya;
3. menjelaskan aturan sinus dan kosinus;
4. menyelesaikan masalah yang berkaitan dengan aturan sinus dan kosinus;
5. menjelaskan fungsi trigonometri dengan menggunakan lingkaran satuan;
6. membuat sketsa grafik fungsi trigonometri;
7. menganalisa perubahan grafik fungsi trigonometri akibat perubahan pada konstanta pada fungsi y = a sin b(x + c) + d.
Berdasarkan pengetahuan dan keterampilan yang dikuasai, siswa mampu:
1. percaya diri dalam membuktikan identitas trigonometri;
2. berpikir logis dalam menyelesaikan masalah kontekstual;
3. bekerja sama dan bertanggung jawab dalam menghadapi masalah.

Identitas Menjelaskan Identitas Kebalikan, Identitas Per-


Trigonometri bandingan, dan Identitas Pythagoras

Membuktikan Identitas Trigonometri


Latihan 1
Aturan Sinus,
Aturan Kosinus, Membuktikan Aturan Sinus dan Aturan Kosinus.
dan Luas Segitiga
Latihan 2 Menentukan Luas Segitiga jika Diketahui (Dua Sisi
dan Satu Sudut), (Dua Sudut dan Satu Sisi), atau
(Panjang Ketiga Sisi)
Soal-Soal Materi
Latihan 3
Menentukan Nilai Maksimum, Nilai Minimum,
Amplitudo, Periode, Daerah Asal, dan Daerah Hasil
Grafik Fungsi Grafik Fungsi Trigonometri
Ulangan Harian Trigonometri
Menggambar Grafik Fungsi Trigonometri Mengguna-
kan Lingkaran Satuan

Menggambar Grafik Fungsi Trigonometri Mengguna-


kan Tabel Titik Bantu
Trigonometri
Membuktikan Identitas Kebalikan, Identitas Per-
(2) bandingan, dan Identitas Pythagoras Berlaku untuk
Sudut Sembarang.

Tugas Menunjukkan Rumus Luas Segitiga (Jika Diketahui


Dua Sisi dan Satu Sudut) Berlaku untuk Segitiga
Sembarang
Penggunaan
Aplikasi Trigono- Membuat Sketsa Grafik Fungsi Trigonometri dari
metry Unit Circle Grafik Fungsi Trigonometri Sederhana
pada Handphone
untuk Menentukan Membuktikan Identitas Trigonometri
Nilai Trigonometri Pemantapan
Informasi Kegiatan Membuktikan Aturan Sinus dan Kosinus

Menyelidiki Perubahan Koefisien terhadap Perubahan


Grafik Fungsi Trigonometri

Berselancar Mempelajari Lebih Lanjut Aturan Sinus dan Kosinus


Internet
Menggambar Grafik Fungsi Trigonometri

Menggunakan Kalkulator Grafik Online

Matematika Kelas X 23
A. Pilihan Ganda cos2 x
=
cos x
1. Jawaban: c
= cos x
cos x
cotan x sin x 1 Jadi, (sec x + tan x)(1 – sin x) = cos x.
= cos x = . Pilihan a salah.
cos x sin x 5. Jawaban: c
1
Oleh karena sin2 x + cos2 x = 1 maka
sec x
= cos x
=
1
. Pilihan b salah. sin2 x – cos2 x = 1 salah.
sin x
tan x
cos x
sin x Oleh karena 1 + tan2 x = sec2 x
cos x
⇔ tan2 x – sec2 x = –1 maka tan2 x – sec2 x = 1
cos x
= = sin x. Pilihan c benar. salah dan tan2 x + sec2 x = 1 salah.
cotan x cos x
sin x Oleh karena 1 + cotan2 x = cosec2 x
cos x cos x
⇔ 1 = cosec2 x – cotan2 x maka
= = cos2 x. Pilihan d salah. cosec2 x – cotan2 x = 1 benar dan
sec x 1
cos x
cosec2 x + cotan2 x = 1 salah.
cos x
Jadi, pernyataan yang benar pada pilihan c.
cotan x sin x cos2 x 6. Jawaban: c
= 1 = . Pilihan e salah.
secan x sin x
cos x
sin x + tan x
Jadi, identitas trigonometri yang benar adalah 1 + cos x
cos x
sin x = . sin x + sin x
cotan x = cos x
1 + cos x
2. Jawaban: b
sin2 x + sin2 x cotan2 x sin x cos x + sin x

= sin2 x (1 + cotan2 x) = cos x


1 + cos x
= sin2 x (cosec2 x)
= sin2 x (cosec x) 2 sin x(1 + cos x)
=
cos x (1 + cos x)
1
= sin2 x ( )2 sin x
sin x =
cos x
1
= sin2 x × =1
sin2 x = tan x
Jadi, bentuk sederhana dari sin2 x + sin2 x cotan2 x
adalah 1. sin x + tan x
Jadi, bentuk sederhana dari adalah tan x.
1 + cos x
3. Jawaban: a
(sin x + cos x) (sin x – cos x) 7. Jawaban: d
= sin2 x – cos2 x cosec P cos P
1

= sin2 x – (1 – sin2 x) cotan P – cos P = sin P – sin P


cos P
= sin2 x + sin2 x – 1
cos P 1
= 2 sin2 x – 1 = sin P – sin P cos P
Jadi, (sin x + cos x) (sin x – cos x) = 2 sin2 x – 1.
cos2 P 1
4. Jawaban: a = – sin P cos P
sin P cos P
(sec x + tan x)(1 – sin x)
1 sin x cos2 P − 1 −sin2 P
=( + )(1 – sin x) = =
cos x cos x sin P cos P sin P cos P
1 + sin x −sin P
=( )(1 – sin x) = cos P
cos x
(1 + sin x)(1 − sin x) = –tan P
=
cos x cosec P
Jadi, bentuk sederhana cotan P – cos P adalah
1 − sin2 x
=
cos x –tan P.

24 Trigonometri (2)
8. Jawaban: b Ruas kiri:
2 sin A cos A 2 sin A cos A (cosec x + cotan x)(1 – cos x)
=
1 + cos2 A − sin2 A (sin2 A + cos2 A) + cos2 A − sin2 A 1 cos x
=( + )(1 – cos x)
sin x sin x
2 sin A cos A
= 1 + cos x
2 cos2 A =( )(1 – cos x)
sin x
sin A 1 − cos x 2
sin2 x
= = =
cos A sin x sin x
= tan A = sin x = ruas kanan (terbukti)
2 sin A cos A
Jadi, dapat dibuktikan (cosec x + cotan x)
Jadi, bentuk ekuivalen dengan (1 – cos x) = sin x
1 + cos2 A − sin2 A
tan A. 2. a. Membuktikan tan2 y – sin2 y = tan2 y sin2 y
9. Jawaban: c Ruas kiri:
tan2 y – sin2 y
1 1 (1 − sin α) + (1+ sin α)
+ = sin2 y
1+ sin α 1 − sin α (1 + sin α )(1 − sin α) = – sin2 y
cos2 y
2 2
= = sin2 y − sin2 y cos2 y
1 − sin2 α cos 2 α =
cos2 y
1
=2× ( cos α )2 sin2 y(1 − cos2 y)
= cos2 y
= 2 sec2 α
sin y sin2 y
2

1 1 = cos2 y
Jadi, bentuk + ekuivalen dengan
1+ sin α 1 − sin α
2 sec2 α. sin2 y
= cos2 y × sin2 y
10. Jawaban: b = tan2 y sin2 y = ruas kanan
sin C sin C Jadi, dapat dibuktikan tan2 y – sin2 y = tan2 y
+ 1 − cos C
1 + cos C sin2 y.
sin C (1 − cos C) + sin C (1 + cos C) b. Membuktikan (1 – sin2 y)2 + (1 – cos2 y)2 =
=
(1 + cos C)(1 − cos C) 1 – 2 sin2 y cos2 y.
sin C (1 − cos C + 1+ cos C)
Ruas kiri:
= (1 – sin2 y)2 + (1 – cos2 y)2
1 − cos2 C
= (1 – 2 sin2 y + sin4 y) + (1 – 2 cos2 y + cos4 y)
2 sin C 2 = 1 + 1 – 2 (sin2 y + cos2 y) + sin4 y +cos4 y
= =
sin2 C sin C
= 1 + 1 – 2 + sin4 y +cos4 y
= 2 cosec C = sin4 y + cos4 y
sin C sin C = (sin2 y + cos2 y)2 – 2 sin2 y cos2 y
Jadi, bentuk 1 + cos C + 1 − cos C ekuivalen = 12 – 2 sin2 y cos2 y
dengan 2 cosec C. = 1 – 2 sin2 y cos2 y
= ruas kanan
B. Uraian Jadi, dapat dibuktikan (1 – sin2 y)2 + (1 – cos2 y)
= 1 – 2 sin2 y cos2 y.
1. a. Membuktikan sin x cosec x – sin2 x = cos2 x.
Ruas kiri: sin A 1 + cos A
3. a. Membuktikan 1 − cos A
= sin A
sin x cosec x – sin2 x
Ruas kiri:
1
= sin x × – sin2 x sin A sin A 1 + cos A
sin x = ×
=1– x sin2 1 − cos A 1 − cos A 1 + cos A
= cos2 x sin (1 + cos A)
=
= ruas kanan (terbukti) 1 − cos2 A
Jadi, dapat dibuktikan sin x cosec x – sin2 x sin (1 + cos A)
=
= cos2 x. sin2 A

b. Membuktikan (cosec x + cotan x)(1 – cos x) 1 + cos A


= sin A
= ruas kanan
= sin x.
sin A 1 + cos A
Jadi, dapat dibuktikan = 1 − cos A
= sin A
.

Matematika Kelas X 25
b. Membuktikan sin A + cos A cotan A = cosec A 2 sin2 α − 1
Ruas kiri: 5. a. Membuktikan = tan α – cotan α.
sin α − cos α
sin A + cos A cotan A Ruas kiri:
cos A
= sin A + cos A × 2 sin2 α − 1
sin A
sin α − cos α
cos2 A
= sin A + 2 sin2 α − (sin2 α + cos2 α)
sin A =
sin α cos α
sin2 A + cos2 A
= 2 sin2 α − sin2 α − cos2 α
sin A
=
1 sin α cos α
= sin A sin2 α − cos2 α
= cosec A = ruas kanan =
sin α cos α
Jadi, dapat dibuktikan sin A + cos A cotan A sin2 α cos2 α
= cosec A. = –
sin α cos α sin α cos α
sin A
c. Membuktikan cosec A + cotan A = 1 − cos A
sin α cos α
= cos α – sin α
Ruas kiri:
cosec A + cotan A = tan α – cotan α
= ruas kanan (terbukti)
1 cos A
= +
sin A sin A 2 sin2 α − 1
Jadi, dapat dibuktikan = tan α
1 + cos A sin α − cos α
= sin A – cotan α.
1 + cos A 1 − cos A b. Membuktikan
= ×
sin A 1 − cos A
sec 2 α − sec α tan α 1
1 − cos2 A =
= cos2 α cos2 α (1 + sin α)
sin A(1 − cos A)
Ruas kiri:
sin2A
=
sin A(1 − cos A) sec 2 α − sec α tan α
sin A cos2 α
= = ruas kanan
1 − cos A
sec 2 α − sec α tan α
Jadi, dapat dibuktikan cosec A + cotan A = =
cos2 α
sin A
1 − cos A
. 1
− 1
× sin α
cos2 α cos α cos α
4. a. (sin B + cos B)2 + (sin B – cos B)2 =
cos2 α
= sin2 B + 2 sin B cos B + cos2 B + sin2 B
1 − sin α
– 2 sin B cos B + cos2 B
cos2 α
= 2 sin2 B + 2 cos2 B =
cos2 α
= 2(sin2 B + cos2 B)
= 2×1=2 1 − sin α 1 + sin α
= ×
Jadi, bentuk sederhana (sin B + cos B)2 + cos4 α 1 + sin α
(sin B – cos B)2 adalah 1. 1 − sin2 α
b. (cosec C – cotan C)(1 + cos C) =
cos 4 α(1 + sin α)
= cosec C – cotan C + cosec C cos C cos2 α
– cotan C cos C = 4
cos α(1 + sin α)
1 cos C 1 cos C
= sin C
– sin C + sin C × cos C – sin C × cos C =
1
cos2 α(1 + sin α)
1 cos C cos C cos2 C
= sin C
– sin C + sin C – = ruas kanan (terbukti)
sin C
1 − cos C2 sec 2 α − sec α tan α
= Jadi, dapat dibuktikan =
sin C cos2 α
sin2 C 1 .
= = sin C
sin C cos2 α (1 + sin α)
Jadi, bentuk sederhana (cosec C – cotan C)
(1 + cos C) adalah sin C.

26 Trigonometri (2)
A. Pilihan Ganda QR 464 2
⇔ 1 = 1
1. Jawaban: a 2 2
2
Bentuk segitiga PQR R
⇔ Q R = 464
dan ukurannya disaji-
kan seperti gambar di Jadi, panjang QR = 464 m.
samping. Pada segitiga 4. Jawaban: c
PQR berlaku aturan 60° 45° Perhatikan ΔABD.
sinus. P Q
AD BD
QR PR = sin A
sin ∠ABD
sin P = sin Q
2 BD
sin P
⇔ sin 30°
= sin 45°
QR
⇔ PR
= sin Q
2 BD
⇔ 1 = 1
QR sin 60° 2 2
2
⇔ PR
= sin 45°
⇔ BD = 2 2
QR
1
2
3 Perhatikan ΔBCD.
⇔ PR
= 1
2
2 BD BC
sin C
= sin ∠BDC
QR 3
⇔ = 2 2 4
PR 2 ⇔ = sin 60°
sin C
Jadi, perbandingan antara panjang QR dengan
2 2 4
PR adalah 3 : 2. ⇔ = 1
sin C 3
2
2. Jawaban: c 1
(2 2)( 2 3)
Aturan sinus: ⇔ sin C =
4
AC BC
= 1
sin B s in A
⇔ sin C = 4 6
AC 4 2
⇔ sin 30°
= sin 45° 1
Jadi, sin C = 4 6 .
AC 4 2
⇔ 1 = 1 5. Jawaban: b
2
2
2
Perhatikan ΔADC.
⇔ AC = 4
Jadi, panjang AC = 4 cm. CD = AC2 − AD2
3. Jawaban: b
R
= 62 − 42

= 36 − 16
∠R = 180° – (105° + 30°)
= 45° = 20
30° 105°
P 464 2 m Q =2 5
Perhatikan ΔBDC.
Berlaku aturan sinus:
CD
QR PQ tan B = BD
=
sin P sin R
2 5
⇔ tan 30° =
QR 464 2 BD
⇔ sin 30°
= sin 45° 1 2 5
⇔ 3 =
3 BD

Matematika Kelas X 27
2 5 9
⇔ BD = 1
⇔ –cos K = 40
3
3
9
2 5 3 ⇔ cos K = – 40
⇔ BD = 1
×
3 3
3 Panjang sisi LM ditentukan dengan aturan
kosinus, diperoleh:
⇔ BD = 2 15 LM2 = KL2 + KM2 – 2 × KL × KM × cos K
9
AB = AD + BD = 4 + 2 15 = 82 + 102 – 2 × 8 × 10 × (– 40 )
Perhatikan ΔABC. = 64 + 100 + 36
AC AB = 200
sin B
= sin ∠ACB LM2 = 200
6 4 + 2 15 ⇔ LM = 200 = 10 2
⇔ =
sin 30° sin ∠ACB
Jadi, panjang sisi LM = 10 2 cm.
6 4 + 2 15
⇔ = 8. Jawaban: c
1
sin ∠ACB
2
1
sin θ = 3
2 + 15
⇔ sin ∠ACB = C
6
2
2 + 15
Jadi, nilai sin ∠ACB = .
6 A θ B
6. Jawaban: c 2 2
Pada segitiga ABC berlaku aturan sinus. Diperoleh:
cos2 θ = 1 – sin2 θ
AB AC 10 12
= ⇔ = 1
sin C sin B sin C 4
5
⇔ cos2 θ = 1 – 9
⇔ 12 sin C = 8 8
⇔ cos2 θ = 9
8 2
⇔ sin C = 12
= 3 2
8
⇔ cos θ = = 3 2
2 2 9
p= 3 −2
= 9−4 AC2 = AB2 + BC2 – 2AB × BC × cos θ
3
2 2
= 5 ⇔ AC2 = (2 2 )2 + 22 – 2 × 2 2 × 2 × 3 2

p 5 C p 2
cos C = = ⇔ AC2 = 8 + 4 – 8 2 × 3 2
3 3
32
Jadi, nilai cos C =
1
5. ⇔ AC2 = 12 – 3
3
4
7. Jawaban: c ⇔ AC2 = 3
Bentuk segitiga KLM M 2
4 3 2
dan ukurannya disaji- ⇔ AC = = 3
× = 3 3
3 3
kan seperti gambar di
2
samping. Jumlah Jadi, panjang AC = 3 3 .
10 cm
ketiga sudut pada
8 cm L 9. Jawaban: e
segitiga KLM adalah K
Gambar di atas dapat
180°, diperoleh: D
dilengkapi seperti gambar
∠K + ∠L + ∠M = 180° 2 C
di samping. Segi empat α
⇔ ∠L + ∠M = 180° – ∠K
ABCD merupakan segi 8
9 4
cos (L + M) = 40
empat tali busur se-
9 hingga ∠A + ∠C = 180°
⇔ cos (180° – K) = 40 ⇔ ∠A = 180° – LC. A 6 B

Perhatikan segitiga ABD.

28 Trigonometri (2)
BD2 = AB2 + AD2 – 2 × AB × AD × cos A Panjang sisi segi-12 ditunjukkan oleh garis AB.
= 62 + 82 – 2 × 6 × 8 × cos A Besar sudut AOB:
= 36 + 64 – 96 × cos A 360°
∠AOB = 12
= 30°.
= 100 – 96 cos A . . . (1)
Perhatikan segitiga BCD. Pada segitiga AOB berlaku aturan kosinus.
BD2 = BC2 + CD2 – 2 × BC × CD × cos C AB2 = AO2 + BO2 – 2 × AO × BO × cos ∠AOB
= 42 + 22 – 2 × 4 × 2 × cos C = 62 + 62 – 2 × 6 × 6 × cos 30°
= 16 + 4 – 16 × cos C 1
= 36 + 36 – 72 × 3
= 20 – 16 cos C . . . (2) 2
Dari persamaan (1) dan (2) diperoleh: = 72 – 36 3
100 – 96 cos A = 20 – 16 cos C = 36(2 – 3)
⇔ 80 – 96 cos A = –16 cos C
AB2 = 36(2 – 3)
⇔ 80 – 96 cos (180° – C) = –16 cos C
⇔ 80 + 96 cos C = –16 cos C ⇔ AB = 36(2 − 3)
⇔ 112 cos C = –80
= 36 2 -- 3
80 5
⇔ cos C = – 112 = – 7
= 6 2 -- 3
Oleh karena besar sudut C adalah α maka nilai Jadi, panjang sisi segi-12 beraturan tersebut
5
cos C = cos α = – 7 . adalah 6 2 -- 3 cm.
5 12. Jawaban: d
Jadi, nilai cos α = – 7
.
Perhatikan gambar berikut.
10. Jawaban: b D x C
AB2 = OA2 + OB2 – 2(OA)(OB) cos α A 120°
120° 10
⇔ 42 = 32 + 32 – 2(3)(3) cos α 3
10
⇔ 16 = 9 + 9 – 18 cos α 60° 60°
A B
⇔ 16 = 18 – 18 cos α α 4
18
O
⇔ –2 = –18 cos α Dalam menentukan panjang AC ada 2 cara
1 3
berikut.
⇔ cos α = 9 AC2 = AB2 + BC2 – 2EB × BC cos ∠B
B
atau
p = 92 − 12 AC2 = AD2 + DC2 – 2AD × DC cos ∠D
9 Dengan menggunakan kedua persamaan
= 81 − 1 p diperoleh sebagai berikut.
α AD2 + DC2 – 2AD × CD cos D
= 80 1 = AB2 + BC2 – 2AB × BC cos ∠B
=4 5 102 + x2 – 2 × 10 × cos 120°
p 4 5
= 182 + 102 – 2 × 18 × 10 cos 60°
tan α = 1 = =4 5 1 1
1
⇔ 100 + x2 – 20x × (– 2 ) = 324 + 100 – 360 × 2
Jadi, nilai tan α = 4 5 . ⇔ x2 + 10x = 324 – 180
11. Jawaban: e ⇔ x2 + 10x = 144
Bentuk lingkaran dan segi-12 beraturan disajikan ⇔ 2
x + 10x – 144 = 0
seperti gambar berikut. ⇔ (x – 8)(x + 18) = 0
⇔ x = 8 atau x = –18
Nilai x yang memenuhi adalah 8.
A Jadi, panjang CD = 8 cm.
m
6c
O 6 cm B

Matematika Kelas X 29
13. Jawaban: a 1
Luas segitiga PQR R
= 8 × 2 × p × p × sin 45°
8 cm
1 120° 1 1
= 2 × PR × QR × sin 120° cm =8× 2 ×p×p× 2 2
15 Q
1 1 = 2p2 2
= 2 × 15 × 8 × 2 3
P Jadi, luas segi delapan tersebut 2p2 2 .
= 30 3 cm2
16. Jawaban: d
Jadi, luas ΔPQR adalah 30 3 cm2.
Bentuk segitiga DEF beserta ukurannya disajikan
14. Jawaban: d seperti di bawah ini.
Bentuk segitiga PQR R
F
beserta ukurannya
disajikan seperti gambar 8 cm 17 cm 25 cm
di samping. L = 24 cm2
Oleh karena luas
segitiga PQR = 24 cm2, P 12 cm Q D E
diperoleh: Oleh karena kelilingnya 70 cm, diperoleh:
LPQR = 24 DE + EF + DF = 70
1 ⇔ DE + 25 + 17 = 70
⇔ × PQ × PR × sin P = 24
2 ⇔ DE + 42 = 70
1 ⇔ DE = 70 – 42
⇔ × 12 × 8 × sin P = 24
2 ⇔ DE = 28
⇔ 48 × sin P = 24 s = setengah keliling = 35 cm
1 Luas segitiga DEF
⇔ sin P = 2
= s(s − DE)(s − EF)(s − DF)
a= 22 − 12
= 35(35 − 28) × (35 − 25) × (35 − 17)
= 4 −1 2
1 = 35 × 7 × 10 × 18
= 3
P a
= 35 × 7 × 5 × 2 × 18
1 1
tan P = = 3
3 3 = 35 × 35 × 36
1
Jadi, nilai tan P = 3. = 352 × 62
3

15. Jawaban: d = 35 × 6 = 210


Bentuk lingkaran dan Jadi, luas segitiga DEF adalah 210 cm2.
segi-8 beraturan di- A 17. Jawaban: b
sajikan seperti gambar
F E
di samping. Segi-8 p
beraturan tersusun O B
atas delapan segitiga O
A D
yang kongruen. Salah
satu segitiga itu adalah
segitiga AOB dengan
B C
panjang AO = BO = p.
Besar sudut AOB: AB = BC = CD = DE = EF = AF = 7 cm
360° 360°
∠AOB = = 45°. ∠AOB = = 60°
8 6 O
Luas segi-8 Perhatikan segitiga AOB.
= 8 × LAOB AO = BO
1
= 8 × 2 × AO × BO × sin ∠AOB ∠A + ∠B + ∠O = 180°
⇔ 2∠A + 60° = 180°
A B

30 Trigonometri (2)
⇔ 2∠A = 120° LN dihitung menggunakan rumus Pythagoras.
⇔ ∠A = 60° LN2 = KN2 – KL2
= 172 – 152
∠A = ∠B = ∠O = 60° (segitiga sama sisi)
= 289 – 225
sehingga AO = BO = 7 cm = 64
1
LAOB = 2 × AO × BO × sin ∠AOB LN = 64 = 8 cm
1 Bangun segi empat KLMN terdiri atas segitiga
= 2 × 7 × 7 × sin 60° KLN dan LMN sehingga luas segi empat KLMN
dapat ditentukan dengan menjumlah luas kedua
1 1
= 2 ×7×7× 2 3 segitiga tersebut.
1
= 21,19 LΔKLN = 2 × KL × LN
Lsegi enam = 6 × LAOB = 6 × 21,19 = 127,14 cm2 1
= 2 × 15 × 8
22 22
Llingkaran = π × r2 = 7
× AO = 7
× 72 = 154 cm2
= 60 cm2
Larsiran = Llingkaran – Lsegi enam 1
LΔLMN = 2 × LM × LN × sin ∠LNM
= 154 – 127,14
= 26,86 1
= 2 × 8 × 12 × sin 120°
Jadi, luas arsiran 26,86 cm2.
1
18. Jawaban: b = 48 × 2 3
Segitiga SPQ siku-siku di P dengan luas:
1 = 24 3 cm2
LI = 2
× PQ × PS LKLMN = LKLM + LLMN
1
= 2
× 12 × 5 = 30 cm2 = (60 + 24 3 ) cm2

QS = PS2 + PQ2 Jadi, luas segi empat KLMN adalah (60 + 24 3 )


cm2.
= 25 + 144 = 169 = 13 cm
Luas segitiga SQR: 20. Jawaban: e
1
LII = × QS × QR × sin ∠SQR BC2 + AB2 − AC2
2 cos B =
2BC × AB
1
= 2 × 13 × 8 × sin 150° 25 + 25 − 75
= 2×5×5
1 25 1
= 52 × 2 = – 50 = – 2
= 26 cm2 1
cos B=–2
Luas PQRS = LI + LII
= 30 + 26 ⇔ B = 120°
= 56 cm2
Jadi, luas PQRS = 56 cm2. 1
LΔABC = 2 × AB × BC × sin B
19. Jawaban: a 1
Dari soal dapat diperoleh gambar segi empat = 2 × 5 × 5 × sin 120°
KLMN berikut. 25 1
= 2 × 2 3 cm2
N
25
17 cm = 4 3 cm2

K 120° Volume prisma = LΔABC × tinggi


15 cm L
12 cm 25
= 4 3×8
M

= 50 3 cm3
Jadi, volume prisma 50 3 cm3.

Matematika Kelas X 31
B. Uraian 1
× 10
⇔ sin P = 2
AB AC 15
1. a. = C
sin C sin B
60° 5 1
3 2 3 2 3 ⇔ sin P = =
15 3
⇔ =
sin 60° sin B
a = 32 − 12
3 2 3 A 3
⇔ = B 3 1
1
3 sin B = 9 −1
2
1
3×2 3 = 8 =2 2
⇔ sin B = 2
P
3 a a
2 2
⇔ sin B = 1 cos P = 3 =
3
⇔ ∠B = 90° 2
Jadi, nilai cos P = 3 2 .
∠A + ∠B + ∠C = 180°
⇔ ∠A + 90° + 60° = 180° b. QR2 = PQ2 + PR2 – 2 × PQ × PR cos P
⇔ ∠A + 150° = 180° ⇔ 52 = 32 + 72 – 2 × 3 × 7 cos P
⇔ ∠A = 30° ⇔ 25 = 9 + 49 – 42 cos P
AC BC ⇔ –33 = –42 cos P P
sin B
= sin A 33 11
BC
⇔ cos P = 42 = 14 3 7
2 3
⇔ = sin 30°
sin 90° 11
Jadi, nilai cos P = 14 . Q R
2 3 BC 5
⇔ = 1
1
2 3. Bentuk jajargenjang ABCD beserta ukurannya
disajikan seperti gambar berikut.
1
⇔ BC = ×2 3 D
2 C
⇔ BC = 3 cm
4 7 cm
Jadi, ∠A = 30°, ∠B = 90°, ∠C = 60°, AB = 3 cm, 8 cm

BC = 3 cm, dan AC = 2 3 cm.


A 12 cm B
b. ∠A + ∠B + ∠C = 180°
Pada gambar di atas panjang AD = BC = 8 cm.
⇔ 120° + 30° + ∠C = 180°
a. Perhatikan segitiga ABD, berlaku:
⇔ 150° + ∠C = 180°
BD2 = AB2 + AD2 – 2 × AB × AD × cos A
⇔ ∠C = 30°
Oleh karena ∠B = ∠C = 30° (segitiga ABC ⇔ (4 7)2 = 122 + 82 – 2 × 12 × 8 × cos A
sama kaki) sehingga AB = AC = 44 cm. ⇔ 112 = 144 + 64 – 192 × cos A
AB BC ⇔ 112 = 208 – 192 × cos A
= ⇔ 192 cos A = 208 – 112
sin C sin A
44 BC ⇔ 192 cos A = 96
⇔ sin 30°
= sin 120°
1
⇔ cos A =
44 BC 2
⇔ 1 = 1 ⇔ cos A = cos 60°
3
2 2
⇔ A = 60°
⇔ BC = 44 3 Jadi, besar sudut A adalah 60°.
Jadi, ∠A = 120°, ∠B = 30°, ∠C = 30°, AB = b. Pada jajargenjang ABCD, jumlah sudut yang
berdekatan adalah 180°. Dengan demikian:
44 cm, BC = 44 3 cm, dan AC = 44 cm. ∠A + ∠ABC = 180°
QR PQ ⇔ 60° + ∠ABC = 180°
2. a. = P ⇔ ∠ABC = 180° – 60°
sin P sin R
10 15 ⇔ ∠ABC = 120°
⇔ sin P
= sin 30° Panjang diagonal AC ditentukan dengan
15
10 15 aturan kosinus.
⇔ sin P
= 1 30° R
2 Q 10

32 Trigonometri (2)
AC2 = AB2 + BC2 – 2 × AB × BC × cos ∠ABC Perbandingan antara luas segitiga dan persegi
= 122 + 82 – 2 × 12 × 8 × cos 120° 1 2
a 3 3a 2 3 3 3
1 = 4
= =
= 144 + 64 – 2 × 12 × 8 × (– ) 1 2
a 2a 2 2
2 6
= 144 + 64 + 96 Jadi, perbandingan antara luas segitiga dengan
= 304
AC2 = 304 luas persegi adalah 3 3 : 2.
C
⇔ AC = 304 = 16 × 19 = 4 19 5. AB + BC + CA = p
Oleh karena ΔABC
Jadi, panjang diagonal AC = 4 19 cm. sama sisi maka
4. Gambar di atas dapat p F E
AB = BC = CA = 3 Q
disajikan ulang seperti di
samping. Misalkan panjang QD + QE + QF = s
sisi segitiga adalah a dan s
QD = QE = QF = 3 A D B
panjang sisi persegi p
adalah p. p Luas ΔABC = luas ΔAQB + luas ΔBQC
Luas segitiga sama sisi + luas ΔCQA
1
a Oleh karena ΔAQB, ΔBQC, dan ΔCQA saling
= × a × a × sin 60° kongruen maka luas ΔAQB = luas ΔBQC = luas
2
1 1 ΔCQA. Sehingga:
= ×a×a× 2 3 luas ΔABC = 3 × luas ΔAQB
2
1 ⎞
= a2 3 ⇔
1
× AB × CA × sin 60° = 3 ⎜ 2 × AB × QD⎟
⎛1
4
2 ⎝ ⎠
Lingkaran di dalam segitiga disebut dengan
lingkaran dalam. Misalkan panjang jari-jari 1 p p 3 1 p s
⇔ × 3 × 3 × =3× 2 × 3 × 3
lingkaran dalam adalah r, diperoleh: 2 2
1 2
luas segitiga a 3 1 p 3
r = = 4
= a 3 ⇔ = 3s
setengah keliling 1
× 3a 6 2
2

Oleh karena diagonal persegi merupakan diameter 6s 3


⇔ p = 3
×
lingkaran dalam maka panjang diagonal persegi 3

1 ⇔ p = 2s 3
adalah a 3 . Dengan demikian:
3
1 Jadi, p = 2s 3 .
p2 + p2= ( a 3 )2
3
1 2
⇔ 2p2 = a
3
1 2
⇔ p2 = a
6
1
Luas persegi = p2 = a2
6

A. Pilihan Ganda π
= 3 tan 4
1. Jawaban: b
=3×1
π
f(x) = 3 tan 2(x – 8 ) =3
π
π π π Jadi, nilai fungsi f(x) = 3 tan 2(x – 8 ) untuk nilai
f( 4 ) = 3 tan 2( 4 – 8 )
π
π x = 4 adalah 3.
= 3 tan 2( 8 )

Matematika Kelas X 33
2. Jawaban: d π
Tabel nilai f(x) = 2 cos x. Grafik fungsi y = tan (x – 2 ).
Y
x y = 2 cos x (x, y) 2 π
y = f(x) = tan (π – 2
)
0 2 (0, 2)
1
π π
3 (6 , 3)
6 X
π
0 π π 3π 2π 5π
π
2 (4 , 2) 2 2 2
4 –1
π π
1 ( 3 , 1)
3
–2
π π
0 ( 2 , 0)
2
2π 2π Catatan:
–1 ( 3 , –1)
3
π

– 2

( 4 ,– 2) Grafik fungsi y = tan (x – 2 ) dapat diperoleh
4
5π 5π dengan menggeser grafik fungsi y = tan x
– 3 ( 6 ,– 3)
6
π
π –2 (π, –2) (digambar putus-putus) ke kanan sejauh 2 satuan
4π 4π searah sumbu X.
–1 ( 3 , –1)
3
π

0

( 2 , 90) Y y = f(x) = tan (π – 2
)
2 3
2π 2 (2π, 2) 2
Grafik fungsi f(x) = 2 cos x. 1
Y X
2 0 π π 3π 2π 5π 3π
–1 2 2 2

1 y = f(x) = 2 cos x
–2
π X
0 π 3π 2π
2
–1 2 Jadi, grafik yang sesuai pada pilihan a.
–2 4. Jawaban: e
1
Jadi, grafik fungsi f(x) = 2 cos x ditunjukkan oleh Tabel nilai fungsi y = sin 2 x.
gambar pada pilihan d.
1
3. Jawaban: a x y = sin x (x, y)
2
π
Tabel nilai fungsi y = tan (x – 2 ). 0 0 (0, 0)
π 1 π 1
x y = tan x (x, y) 2
(3, 2
)
3

π π π 2 π 2
0 ( 2 , 0) (2, )
2 2 2 2
π π
6
– 3 ( 6
,– 3) π 1 (π, 1)
π π 3π 2 3π 2
–1 ( 4 , –1) ( 2 , )
4 2 2 2
π 3 π 3 5π 1 5π 1
– (3, – ) ( 3 , )
3 3 3 3 2 2
π π
– ( 2 , –) 2π 0 (2π, 0)
2
2π 3 2π 3
( 3 , ) 3π –1 (3π, –1)
3 3 3
3π 3π
1 ( 4 , –1) 4π 0 (4π, 0)
4
5π 5π
3 ( 6 , 3)
6

π 0 (π, 0)
4π 3 4π 3
– ( 3 , )
3 3 3
3π 3π
0 ( 2 , 0)
2

34 Trigonometri (2)
1 Cara lain:
Grafik fungsi y = sin 2 x: Untuk y = 0:
Y π
2
y = sin (x – 2 )
1
y = sin 2
x
1 π
⇔ 0 = sin (x – 2 )
X
0 π 2π 3π 4π
–1 π
⇔ sin (x – 2 ) = 0
–2
π π
Jadi, grafik yang sesuai pada pilihan e. ⇔ sin (x – 2 ) = sin 0 atau sin (x – 2 ) = sin π

5. Jawaban: c π
1) sin (x – 2 ) = sin 0
Fungsi y = a sin b(x – c) + d mempunyai nilai
maksimum ymaks = |a| + d dan nilai minimum π
⇔ (x – 2 ) = 0 + k × π
ymin = –|a| + d.
π
Untuk y = 3 sin 4x maka |a| = 3 dan d = 0. ⇔ x= 2 +k×π
Nilai maksimum ymaks = |a| + d = 3 + 0 = 3. π π
Untuk k = 0 ⇒ x = 2 + 0 × 2π = 2
Jadi, nilai maksimum y = 3 sin 4x adalah 3.
6. Jawaban: a π
2) sin (x – 2 ) = sin π
π
Grafik fungsi y = cos 2(x – 2 ) – 3 memotong π
⇔ (x – 2 ) = π + k × 2π
sumbu Y jika x = 0.
Untuk x = 0: 3π
⇔ x = 2 + k × 2π
π
y = cos 2(0 – 2 ) – 3 3π 3π
Untuk k = 0 ⇒ x = 2 + 0 × 2π = 2
π
= cos 2 (– 2 ) – 3 π
Jadi, titik potong grafik fungsi y = sin (x – 2 )
= cos (–π) – 3
π 3π
= –1 – 3 dengan sumbu Y adalah ( 2 , 0) dan ( 2 , 0).
= –4
π 8. Jawaban: e
Jadi, titik potong grafik fungsi y = cos 2(x – 2 ) – 3 Tabel nilai fungsi y = 5 – 2 cos 3x.
dengan sumbu Y adalah (0, –4).
x y = 5 – 2 cos 3x (x, y)
7. Jawaban: e
0 3 (0, 3)
π
Grafik fungsi y = sin (x – 2 ) memotong sumbu X π
5
π
( 6 , 5)
6
jika y = 0. π π
7 ( 3 , 7)
3
π
Perhatikan grafik fungsi y = sin (x – 2
) berikut. π
5
π
( 2 , 5)
2
Y 2π 2π
3 ( 3 , 3)
3
1 y = sin (x –
π
)
2 π 7 (π, 7)
7π 7π
X 5 ( 6 , 5)
0 π 6
π 3π 2π
2 2 4π 4π
3 ( 3 , 3)
3
–1 3π 3π
7 ( 2 , 7)
π 2
Grafik tersebut memotong sumbu X di titik (2 , 0)
2π 3 (2π, 3)

dan ( 2 , 0).
π
Jadi, titik potong grafik fungsi y = sin (x – 2 )
π 3π
dengan sumbu Y adalah ( 2 , 0) dan ( 2 , 0).

Matematika Kelas X 35
Grafik fungsi y = 5 – 2 cos 3x. π
Grafik fungsi y = 2 sin 3(x – 6 ).
Y
8 Y
2
6 y = 5 – 2 cos 3x π
y = 2 sin 3(x – 6
)
1
4
2π 4π
−π π π X
2 3 3 0 5π 7π 3π 11π
6 6 2 6 6 2 6
–1
0 X
π π π 2π 5 π π 7π 4π 3π 5π 11π 2π
6 3 2 3 6 6 3 2 3 6 –2
Daerah hasil fungsi y = 5 – 2 cos 3x selalu terletak π
di antara 3 dan 7. Jadi, grafik y = 2 sin 3(x – 6 ) ada pada gambar
Jadi, daerah hasil fungsi y = 5 – 2 cos 3x adalah pilihan e.
{y | 3 ≤ y ≤ 7}.
10 Jawaban: b
Cara lain: π
Fungsi y = cos 3x mempunyai nilai maksimum 1 Tabel nilai fungsi y = 2 – cos 2(x – 2 ).
dan nilai minimum 2.
π
Untuk cos 3x = 1: x y = 2 – cos 2(x – 2 ) (x, y)
y = 5 – 2 cos 3x = 5 – 2 × 1 = 5 – 2 = 3 (minimum)
0 3 (0, 3)
Untuk cos 3x = –1:
π π
y = 5 – 2 cos 3x 4
2 ( 4 , 2)
= 5 – 2 × (–1) π π
1 ( 2 , 1)
= 5 + 2 = 7 (maksimum) 2
3π 3π
Nilai fungsi y = 5 – 2 cos 3x terletak di antara 3 4
2 ( 4 , 2)
dan 7.
π 3 (π, 3)
Jadi, daerah hasil fungsi y = 5 – 2 cos 3x adalah
5π 5π
{y | 3 ≤ y ≤ 7}. 2
1 ( 2 , 1)
3π 3π
9. Jawaban: e 1 ( 2 , 1)
2
π 7π 7π
Tabel nilai fungsi y = 2 sin 3(x – 6 ). 4
2 ( 4 , 2)

π 2π 3 (2π, 3)
x y = 2 sin 3(x – ) (x, y)
6
π
0 –2 (0, –2) Grafik fungsi y = 2 – cos 2(x – 2 ).
π π
0 ( 6 , 0) Y
6
3
π π
2 (3, 2)
3
π π 2
0 (2, 0)
2
2π 2π
–2 ( 3 , –2) 1
3

π 2 (π, 2) X
0 π π 3π 2π
7π 7π 2
0 ( 6 , 0) 2 2
6
4π 4π
π
–2 ( 3 , –2) Jadi, grafik fungsi y = 2 – cos 2(x – 2 ) ditunjukkan
3

0

( 2 , 0)
pada pilihan b.
2
5π 5π
2 ( 3 , 2)
3

2π –2 (2π, –2)

36 Trigonometri (2)
B. Uraian Y
3
1. a. Tabel nilai fungsi y1 = sin x dan y2 = cosec x.
2
y3 = cos x
x y1 = sin x y2 = cosec x 1 y4 = sec x

0 1 (0, 1) ~ X
0 π π 3π π 5π 3π 7π 2π
π π
–1 4 2 4 4 2 4
π 1 1
2
(6, 2
) 2 ( 6 , 2) –2
6
π 1 π 1 π
2 2 (4 , 2 2) 2 (4 , 2)
4
π 1 π 1 2 3 π 2 3
3 2 3 (3 , 2 3) 3
(3 , 3
) c. Tabel nilai fungsi y5 = tan x dan y6 = cotan x.
π π π
1 (2 , 1) 1 (2 , 1) x y3 = tan x y4 = cotan x
2

π 0 (π, 0) ~ 0 0 (0, 0) ~
3π 2 3π 2 3π π 3 π 3 π
( 4 , ) 2 ( 4 , 2) (6, ) 3 (6, 3)
4 2 2 6 3 3
5π 1 5π 1 5π π π π
– 2 ( 4 ,– 2) – 2 ( 4 ,– 2) 1 (4 , 1) 1 (4 , 1)
4 2 2 4
3π 3π 3π π π 3 π 3
–1 ( 2 , –1) –1 ( 2 , –1) 3 (3 , 3) (3 , )
2 3 3 3
7π 1 7π 1 7π π π
– 2 ( 4 ,– 2) – 2 ( 4 ,– 2) ~ 0 (2 , 0)
4 2 2 2

2π 0 (2π, 0) ~ π 0 (π, 0) ~
5π 5π 5π
1 ( 4 , 1) 1 ( 4 , 1)
Y 4
3π 3π
4 ~ 0 ( 2 , 0)
2
3 7π 7π 7π
2 –1 ( 4 , –1) –1 ( 4 , –1)
y1 = sin x 4
1 2π 0 (2π, 0) ~
0 π X
π π 3π 5π 3π 7π 2π
–1 4 2 4 4 2 4
–2 x
y2 = cosec x tan an
x
–3 Y y= cot
y =
–4 4

b. Tabel nilai fungsi y3 = cos x dan y4 = sec x. 2


x y3 = cos x y4 = sec x
X
0 1 (0, 1) 1 (0, 1) 0 π π 3π π 5π 3π 7π 2π
4 2 4 4 2 2
π 1 π 1 2 3 π 2 3
3 (6, 3) (6, ) –2
6 2 2 3 3
π 1 π 1 π
2 (4, 2) 2 (4, 2)
4 2 2
π 1 π 1 π 2. a. Fungsi f(x) = sin 4x.
(3 , ) 2 (3 , 2)
3 2 2
π π
1) Grafik fungsi memotong sumbu Y jika
0 (2 , 0) ~ x = 0.
2

π –1 (π, –1) –1 (π, –1) f(0) = sin 0 = 0


5π 1 5π 1 5π
Grafik fungsi memotong sumbu Y di titik
– 2 ( 4 ,– 2) – 2 ( 4 ,– 2) (0, 0).
4 2 2

0

( 2 , 0) ~ Grafik fungsi memotong sumbu X jika
2
7π 1 7π 1 7π
y = 0 atau f(x) = 0.
4 2 2 ( 4 , 2 2) 2 ( 4 , 2) f(x) = 0 ⇔ sin 4x = 0
2π 1 (2π, 1) 1 (2π, 1) ⇔ sin 4x = sin 0 atau sin 4x = sin π
⇔ 4x = 0 + k × 2π atau 4x = π + k × 2π
π π π
⇔ x = 0 + k × 2 atau x = 4 + k × 2
π 3π
⇔ x= 2,π, 2
, 2π , . . .
π 3π 5π 7π 9π
atau x = 4 , 4 , 4 , 4 , 4 , . . . .

Matematika Kelas X 37
Grafik fungsi memotong sumbu X di titik Jadi, titik potong dengan sumbu koordinat
π π 3π 5π 3π π
(4 , 0), (2 , 0), ( 4 , 0), (π, 0), ( 4 , 0), adalah (0, 1) dan ( 8 , 0), ( 8 , 0),
7π 5π 7π 9π 11π
( 4 , 0). ( 8 , 0), ( 8 , 0), ( 8 , 0), ( 8 , 0),
Jadi, titik potong dengan sumbu koordinat
13π 15π
π π 3π ( 8 , 0), ( 8 , 0).
adalah (0, 0), ( 4 , 0), ( 2 , 0), ( 4 , 0),
3π 5π 7π 2) Untuk sembarang nilai x, fungsi g(x) =
(π, 0), ( 4 , 0), ( 4 , 0), ( 4 , 0). cos 4x bernilai –1 ≤ g(x) ≤ 1.
2) Untuk sembarang nilai x, fungsi f(x) = Jadi, nilai maksimum fungsi g(x) = cos
sin 4x bernilai –1 ≤ f(x) ≤ 1. 4x adalah 1 dan nilai minimumnya –1.
Jadi, nilai maksimum fungsi f(x) = sin 4x 3) Daerah hasil fungsi g(x) = cos 4x adalah
adalah 1 dan nilai minimumnya –1. {g(x) |–1 ≤ g(x) ≤ 1, g(x) ∈ R}.
3) Daerah hasil fungsi f(x) = sin 4x adalah 3. a. Sketsa grafik y = 2 sin (x + 30°) dapat dibuat
{f(x) |–1 ≤ f(x) ≤ 1, f(x) ∈ R}. dengan cara menggambar grafik fungsi y1 =
b. Fungsi g(x) = cos 4x. sin x kemudian menggeser ke kiri grafik
fungsi y1 = sin x sejauh 30° searah sumbu X
1) Grafik fungsi memotong sumbu Y jika
sehingga dihasilkan grafik fungsi y2 = sin
x = 0.
(x + 30°). Langkah terakhir, mengalikan
g(0) = cos 0 = 1
semua ordinat dengan 2 pada grafik fungsi
Grafik fungsi memotong sumbu Y di titik
y2 = sin (x + 30°) agar diperoleh grafik fungsi
(0, 1).
y3 = 2 sin (x + 30°).
Grafik fungsi memotong sumbu X jika
y = 0 atau f(x) = 0. Y y = 2 sin (x + 30°)
2
π
⇔ cos 4x = cos 2
y = sin (x + 30°)
1 y1 = sin x
π 180° 330°
⇔ cos 4x = cos (± 2 ) 360°
X
0 150°
π –1
a) cos 4x = cos 2
π –2
⇔ 4x = 2 + k × 2π
π π b. Sketsa grafik y = cos 2x + 2 dapat dibuat
⇔ x= 8 +k× 4 dengan cara menggambar grafik fungsi
y1 = cos x kemudian memampatkan grafik
π 3π 5π 7π
⇔ x= 8, 8
, 8
, 8 ,.... fungsi y 1 = cos x sehingga mempunyai
periode 180° agar dihasilkan grafik fungsi
π
b) cos 4x = cos (– 2 ) y2 = cos 2x. Langkah terakhir, menambahkan
2 pada semua ordinat grafik fungsi y2 = cos
π
⇔ 4x = (– 2 ) + k × 2π 2x agar diperoleh grafik fungsi y = cos 2x + 2.
π π Y
⇔ x = (– 8 ) + k × 4 3
π 3π 5π 7π 9π 11π
⇔ x= 8, 8 , 8 , 8 , 8 , 8 , 2
y = cos 2x + 2
13π 15π 1
y1 = cos x
, 8 ,....
8
y2 = cos 2x

Grafik fungsi memotong sumbu X di titik 0 X


90° 180° 270° 360°
π 3π 5π 7π 9π –1
( 8 , 0), ( 8 , 0), ( 8 , 0), ( 8 , 0), ( 8 , 0),

11π 13π 15π


( 8 , 0), ( 8 , 0), ( 8 , 0).

38 Trigonometri (2)
c. Sketsa grafik y = –sin 2(x – 45°) dapat dibuat 1) Sebagai fungsi sinus.
dengan cara menggambar grafik fungsi Y
y1 = sin 2x kemudian menggeser ke kanan 4 y = sin (x + 90°) + 3
grafik fungsi y1 = sin 2x sejauh 45° searah
sumbu X sehingga dihasilkan grafik fungsi
y2 = sin 2(x – 45°) memampatkan grafik fungsi.
Langkah terakhir, mengalikan semua ordinat
x
dengan (–1) pada grafik fungsi y 3 = sin sin
2(x – 45°) agar diperoleh grafik fungsi y=
X
y4 = –sin 2(x – 45°). 0 90° 180° 270° 360°
) °)
45° 45
x (x – –
in 2 in 2 2(x
Y y1
=s y3 = –s –sin Salah satu titik puncak grafik fungsi
=
1 y2 y = sin x dengan sumbu Y adalah (90°, 1).
Perhatikan titik tersebut bergeser sejauh
00
X 90° ke kiri searah sumbu X dan bergeser
90° 180° 270° 360° 2π
45° ke atas sejauh 3 satuan searah sumbuY.
–1 Jadi, persamaan grafik fungsinya adalah
y = sin (x + 90°) + 3.
4. a. Grafik fungsi tersebut mempunyai 2 2) Sebagai fungsi kosinus
kemungkinan persamaan yaitu fungsi sinus
Y
atau fungsi kosinus.
4
1) Sebagai fungsi sinus.
Y

y = sin x

X
0 60° 240° 360°
X
y = sin (x – 60°) 0 90° 180° 270°

Fungsi f(x) tersebut merupakan fungsi


y = sin x yang digeser ke kanan sejauh Fungsi f(x) tersebut merupakan fungsi
60° satuan searah sumbu X sehingga y = cos x yang digeser ke atas sejauh 3
persamaan grafik fungsi tersebut adalah
satuan searah sumbu Y sehingga
f(x) = sin (x – 60°).
persamaan grafik fungsi tersebut adalah
2) Sebagai fungsi kosinus. f(x) = cos x + 3.
Y 5. Grafik fungsi tersebut mempunyai 2 kemungkinan
1 y = cos x persamaan yaitu fungsi sinus atau fungsi kosinus.
a. Sebagai fungsi kosinus.
X Misalkan persamaan grafik fungsi adalah y
0 60° 150° 240°
π = a cos b(x + c) + d.
y = cos (x – )
3 2π
Fungsi f(x) tersebut merupakan fungsi Grafik fungsi berulang setiap 3 . Nilai
y = cos x yang digeser ke kanan sejauh 2π
150° satuan searah sumbu X sehingga b= 2π = 3.
3
persamaan grafik fungsi tersebut adalah
f(x) = cos (x – 150°) Nilai ymaks = 3 dan nilai ymin = 1. Amplitudo
= cos – (180° – (x + 30°) 1
fungsi = 2 (3 – 1) = 1. Nilai a = 1.
= –cos (x + 30°)
Persamaan fungsi sementara dengan y = cos
b. Grafik fungsi tersebut mempunyai 2
3(x + c) + d.
kemungkinan persamaan yaitu fungsi sinus
atau fungsi kosinus.

Matematika Kelas X 39
Titik potong grafik y = cos 3x sumbu Y yaitu Nilai ymaks = 3 dan nilai ymin = 1. Amplitudo
titik (0, 1) pada tidak mengalami pergeseran fungsi = (3 – 1) = 1. Nilai a = 1.
ke kanan atau ke kiri searah sumbu X Persamaan fungsi menjadi y = sin 3(x + c) + 2.
sehingga c = 0. Akan tetapi titik tersebut Titik potong sumbu Y yaitu (0, 0) pada fungsi
mengalami pergeseran ke atas (+2) searah π
sumbu Y sehingga d = 2. y = sin 3x mengalami pergeseran ke kiri 6
Jadi, persamaan fungsinya y = cos 3(x + 0) searah sumbu X dan pergeseran ke atas +2
+ 2 ⇔ y = cos 3x + 2. π
satuan sehingga c = 6 dan d = 2.
b. Sebagai fungsi sinus.
Misalkan persamaan grafik fungsi y = a sin π
Jadi, persamaan fungsinya y = sin 3(x+ 6 )
b(x + c) + d.
π
2π + 2 ⇔ y = sin 3(x + 6 ) + 2.
Grafik fungsi berulang setiap 3 . Nilai

b= 2π = 3.
3

A. Pilihan Ganda − cos x


= sin x (sin x + 1)
1. Jawaban: b
7 tan2 x + 3 − cos x
=
sin2 x + sin x
= 7(sec2 x – 1) + 3
p − cos x
= 7 sec2 x – 7 + 3 Jadi, = .
q sin2 x + sin x
= 7 sec2 x – 4
7
4. Jawaban: a
= –4 cos2 A + tan2 A – sec2 A
cos 2 x
7
= cos2 A + tan2 A – (1 + tan2 A)
Jadi, 7 tan2x + 3 = – 4. = cos2 A + tan2 A – 1 – tan2 A
cos 2 x
= cos2 A – 1
2. Jawaban: a = cos2 A – (sin2 A + cos2 A)
cos x
−1
cos x − sin x = –sin2 A
cotan x − 1 sin x sin x cos x − sin x
cotan x + 1
= cos x = cos x + sin x = Jadi, cos2 A + tan2 A – sec2 A = –sin2 A.
+1 cos x + sin x
sin x sin x
5. Jawaban: b
cos x − 1 cos x − sin x 1 − tan θ
Jadi, bentuk setara dengan . sin θ + cos θ
cos x + 1 cos x + sin x sin θ − cos θ
× 1 + tan θ
3. Jawaban: c sin θ
sin θ + cos θ 1− cos θ
p tan x −
1
= sin θ − cos θ
× sin θ
= cos x 1+ cos θ
q sin x
cos θ − sin θ
sin x 1 sin θ + cos θ
− = ×
cos θ
cos θ + sin θ
= cos x cos x sin θ − cos θ cos θ
sin x
sin θ + cos θ cos θ − sin θ
sin x − 1 = sin θ − cos θ
×
cos x cos θ + sin θ
=
sin x
sin θ + cos θ −(sin θ − cos θ)
sin x − 1 sin x + 1
= sin θ − cos θ
× = –1
sin θ + cos θ
= ×
sin x cos x sin x + 1
sin θ + cos θ 1 − tan θ
sin2 x − 1 Jadi, nilai × = –1.
= sin θ − cos θ 1 + tan θ
sin x cos x (sin x + 1)
− cos2 x
=
sin x cos x (sin x + 1)

40 Trigonometri (2)
6. Jawaban: e Aturan sinus:
sin α 1 + cos α a b
+ sin A
= sin B
1 + cos α sin α

sin α × sin α (1 + cos α)(1 + cos α) a b


= + ⇔ =
(1 + cos α) sin α (1 + cos α) sin α sin 30° sin 60°

sin2 α 1 + 2 cos α + cos2 α 10 − b b


= + ⇔ 1 = 1
(1 + cos α) sin α (1 + cos α) sin α 2
3
2
2 2
sin α + 1 + 2 cos α + cos α ⇔ 10 3 – b 3 = b
=
(1 + cos α) sin α
⇔ 10 3 = b(1 + 3)
(sin2 α + cos 2 α) + 1 + 2 cos α
= 10 3 1− 3
(1 + cos α) sin α ⇔ b= ×
1+ 3 1− 3
1 + 1 + 2 cos α
= ⇔ b= 10 3 − 30
(1 + cos α) sin α
1− 3
2 + 2 cos α 30 − 10 3
= ⇔ b=
(1 + cos α) sin α 2
2(1 + cos α) ⇔ b = 15 – 5 3
=
(1 + cos α) sin α
Jadi, panjang sisi b = 15 – 5 3 .
2
= sin α = 2 cosec α 9. Jawaban: b
sin α 1 + cos α Aturan kosinus:
Jadi, 1 + cos α
+ sin α
= 2 cosec α. QR2 = PQ2 + PR2 – 2 × PQ × PR cos P
= 92 + 62 – 2 × 9 × 6 cos 60°
7. Jawaban: d 1
∠A + ∠B + ∠C = 180° C = 81 + 36 – 2 × 9 × 6 × 2
⇔ 75° + ∠B + 45° = 180°
45° = 81 + 36 – 54
⇔ ∠B + 120° = 180°
= 63
⇔ ∠B = 60°
Aturan sinus: QR = 63 = 3 7
AB
=
AC 75° Jadi, panjang sisi QR = 3 7 cm.
sin C sin B B
A
10. Jawaban: b
AB AC Untuk menentukan panjang CD diperlukan ukuran
⇔ sin 45°
= sin 60°
panjang BD atau BC. Pada segitiga ABD berlaku
AB AC aturan kosinus sebagai berikut.
⇔ =
1
2
2
1
2
3 BD2 = AB2 + AD2 – 2 × AB × AD × cos A
= 122 + 162 – 2 × 12 × 16 × cos 60°
1
AB 2 1
⇔ = 2
1
= 144 + 256 – 2 × 12 × 16 × 2
AC 3
2
= 144 + 256 – 192
AB 2 = 208
⇔ AC
= BD2 = 208
3
Jadi, perbandingan antara panjang AB dengan AC ⇔ BD = 208 = 4 13
adalah 2 : 3. Segitiga BCD merupakan segitiga siku-siku.
Dengan demikian:
8. Jawaban: c C
a + b = 10 BD 1 4 13
b a tan 30° = ⇔ =
CD CD
⇔ a = 10 – b 3

A
30° 60° ⇔ CD = 4 39
c B
Jadi, panjang CD = 4 39 cm.

Matematika Kelas X 41
11. Jawaban: d Perhatikan segitiga BCD.
B
Ukuran sisi segitiga: BD2 = BC2 + CD2 – 2 × BC × CD × cos C
AB = c = 3 cm 3 cm 7 cm
⇔ 7 = x2 + (2x)2 – 2 × x × 2x × cos 60°
BC = a = 7 cm
A C 1
AC = b = 5 cm 5 cm ⇔ 7 = x2 + 4x2 – 2 × x × 2x × 2
Besar sudut A ditentukan dengan aturan kosinus.
⇔ 7 = x2 + 4x2 – 2x2
a2 = b2 + c2 – 2bc cos A
⇔ 7 = 3x2
⇔ 72 = 52 + 32 – 2 × 5 × 3 × cos A
⇔ 49 = 25 + 9 – 30 cos A 7
⇔ x2 =
⇔ 49 = 34 – 30 cos A 3
⇔ 30 cos A = 34 – 49 7
⇔ 30 cos A = –15 ⇔ x= 3
1
⇔ cos A = – 2 7 3
⇔ x= ×
⇔ cos A = cos 120° 3 3
⇔ A = 120° 1
Jadi, besar sudut A adalah 120°. = 21
3

12. Jawaban: a 1
21 cm.
Jadi, panjang BC =
Dimisalkan panjang sisi segitiga 2a, 3a, dan 4a. 3
Sudut terbesar berada di depan sisi terpanjang, 14. Jawaban: a
yaitu ∠B.
AB2 + BC2 − AC2 C
cos ∠B = 2 × AB × BC A
8 cm
B 30°
(2a)2 + (3a)2 − (4a)2 8 cm
= 2 × 2a × 3a C
4a
4a2 + 9a2 − 16a2 3a
=
12a2
Luas segitiga (s, sd, s):
−3a2 1
= = –4 A B
1
12a2 2a LΔABC = 2 × AB × BC × sin B
1
Jadi, nilai kosinus sudut terbesar adalah – 4 . 1
= 2 × 8 × 8 × sin 30°
13. Jawaban: b 1
Bentuk segi empat ABCD beserta ukurannya = 32 × 2
disajikan seperti di bawah ini.
= 16 cm2
D
2 cm Luas segi-12 beraturan
A
60° = 12 × LΔABC
2x cm = 12 × 16
3 cm = 192 cm2
Jadi, luas segi-12 beraturan tersebut 192 cm2.
60°
C
x cm 15. Jawaban: a
B
B
Misalkan panjang BC = x cm maka panjang 12 cm
CD = 2x cm.
Perhatikan segitiga ABD. A 16 cm C
BD2 = AB2 + AD2 – 2 × AB × AD × cos A Luas segitiga (s, sd, s):
= 32 + 22 – 2 × 3 × 2 × cos 60° 1
L = 2 × AB × AC × sin A
1
=9+4–2×3×2× 2 1
=9+4–6 ⇔ 48 = 2 × 12 × 16 × sin A
=7 ⇔ 48 = 96 sin A
Diperoleh BD2 = 7.

42 Trigonometri (2)
48
⇔ sin A = 96 82 − 72
sin B = 8
1
⇔ sin A = 2 64 − 49
=
8
⇔ sin A = sin 30° 1
= 8 15
⇔ A = 30°
Jadi, besar sudut A = 30°. 1
LΔABC = 2 × AB × BC × sin B
16. Jawaban: d
1 1
Belah ketupat KLMN beserta ukurannya disajikan = 2 × 2a × 2a × 8 15
ulang seperti berikut.
1
N = 4 a2 15 cm2
120° Volume prisma = LΔABC × AD
30° 12 3 30° 1
K M = 4 a2 15 × 4
30° 30°

= a2 15 cm3

L Jadi, volume prisma adalah a2 15 cm3.


Luas segitiga KMN (sd, s, sd): 18. Jawaban: d
Perhatikan ΔABD D C
KM2 × sin 30° × sin 30°
LΔKMN = 2 sin 120° 1
s = 2 (AB + BD + AD)
1 1 14 cm
(12 3)2 × × 1
= 1
2 2 = 2 (6 + 14 + 10) 10 cm
2× 2
3
1
1 1
= 2 × 30
432 × 2
× 2
= = 15
3 A 6 cm B
LABCD = 2 × LABD
108 3 =2× s(s − AB)(s − BD)(s − AD)
= ×
3 3
=2× 15 × (15 − 6) × (15 − 14) × (15 − 10)
= 36 3 cm2
Luas KLMN =2× 15 × 9 × 1 × 5
= 2 × LΔKMN
= 2 × 15 × 45
= 2 × 36 3 cm2
=2× 15 × 15 × 3
Jadi, luas KLMN = 72 3 cm2.
17. Jawaban: c = 2 × 15 3 = 30 3

AB2 + BC 2 − AC 2 Jadi, luas jajargenjang tersebut 30 3 cm2.


cos B = 2AB × BC 19. Jawaban: d
(2a)2 + (2a)2 − a 2 8 1
= 2 × 2a × 2a Amplitudo = 2 (nilai maksimum – nilai minimum)

4a 2 + 4a 2 − a 2 1
= B 7
= 2 (1 – (–1))
8a 2
=1
7a2
= Periode = π, karena untuk setiap interval skala π
8a2
pada sumbu X, grafik akan berulang dengan
7
= 8 bentuk yang sama. Jadi, amplitudo dan periode
grafik tersebut berturut-turut adalah 1 dan π.

Matematika Kelas X 43
20. Jawaban: e π
f(x) = 3 sin (x – 4 ) + 1
π
Grafik fungsi f(x) = sin (x + ) – 1 dapat diperoleh
6 fmin = –|3| + 1 = –3 + 1 = –2.
dengan menggeser grafik f(x) = sin x ke kiri sejauh π
π Jadi, nilai minimum f(x) = 3 sin 2(x – 4 ) + 1 adalah –2.
satuan searah sumbu X kemudian dilanjutkan
6 Cara lain:
menggeser ke bawah sejauh 1 satuan searah π
sumbu Y. Nilai maksimum sin (x – 4 ) adalah 1 dan nilai
π
π ) minimum sin (x – 4 ) adalah –1.
6
+
(x π
sin Untuk sin (x – 4 ) = 1, diperoleh:
Y =
x)
f 2( π
f1(x) = sin x f(x) = 3 sin (x – 4 ) + 1
1
=3×1+4
−π −π 0 π 2π π 11π 2π X = 7 ← (maksimum)
3 6 3
6 6 π
–1 Untuk sin (x – 4 ) = 1, diperoleh:
π π
–2 f3(x) = sin (x + )–1
6 f(x) = 3 sin (x – 4 ) + 1
π = 3 × (–1) + 1
Jadi, grafik fungsi f(x) = sin (x + 6 ) – 1 ditunjukkan = –2 ← (minimum)
oleh gambar pada pilihan e. π
Jadi, nilai minimum f(x) = 3 sin 2(x – 4 ) + 1 adalah –2.
21. Jawaban: b
23. Jawaban: e
π π
Grafik fungsi y = –2 cos (2x – 3 ) = –2 cos 2(x – 6 ) f(x) = a sin b(x + c) + d
dapat diperoleh dengan menggambar grafik y = Nilai f maksimum adalah fmaks = |a| + d dan nilai
cos 2x terlebih dahulu kemudian menggesernya minimum f adalah fmin = –|a| + d.
π π
ke kanan sejauh 6 searah sumbu X agar diperoleh f(x) = 3 sin 2(x – ) + 1
2
π
grafik fungsi y = cos 2(x – 6 ). Grafik fungsi y = –2 f maks = | 3 |+ 1 = 3 +1
π
cos 2(x – 6 ) dapat diperoleh dengan mengalikan fmin = –| 3 |+ 1 = – 3 + 1
π Dengan demikian a = 3 + 1 dan b = – 3 + 1.
ordinat grafik fungsi y = cos 2(x – 6 ) dengan –2
untuk setiap x yang bersesuaian. a2 + b2 = ( 3 + 1)2 + (– 3 + 1)2
= (3 + 2 3 + 1) + (3 – 2 3 + 1)
2x

π
Y π y = cos 2(x – )
=8
s

y = cos 2(x – ) 6
co

2 6
=

Jadi, nilai a2 + b2 = 8.
y

1
5π 5π 23π 24. Jawaban: c
12 4 12
π π 3π 11π 17π
X Fungsi y = 4 sin 2x – 3 (fungsi sinus) mencapai
12 4 4 12 12 nilai minimum pada saat sin 2x = –1.
–1
sin 2x = –1
–2
⇔ sin 2x = sin 270°
⇔ 2x = 270° + k × 360°
π ⇔ x = 135° + k × 180°, k bilangan bulat
Jadi, grafik fungsi y = –2 cos (2x – 3 ) ada pada Jadi, fungsi y = 4 sin 2x – 3 mencapai nilai
pilihan b. minimum pada saat x = 135° + k × 180° dengan k
bilangan bulat.
22. Jawaban: b
f(x) = a sin b(x + c) + d
Nilai f maksimum adalah fmaks = |a| + d dan nilai
minimum f adalah fmin = –|a| + d.

44 Trigonometri (2)
25. Jawaban: a Grafik fungsi kedua dapat diperoleh dengan
πx π
f(x) = 4 + 2 sin akan mencapai minimum ketika menggeser grafik y1 = cos x ke kanan sejauh 3
4
πx satuan searah sumbu X. Dengan demikian
sin mencapai minimum. Oleh karena fungsi
4 persamaan grafik fungsi kedua adalah y2 = cos
sinus mempunyai nilai minimum –1, sehingga:
π
πx πx π (x – 3 ).
sin = –1 ⇔ sin = sin (– )
4 4 2 Jadi, grafik fungsi di atas mempunyai persamaan
πx π π
⇔ =– y = cos (x – 3 ).
4 2
⇔ x = –2
Diperoleh q = x = –2. 28. Jawaban: a
Nilai minimum f(x): Grafik melalui titik (0, –2), diperoleh:
y = a cos kx ⇔ –2 = a cos (k × 0)
πx
fmin = 4 + 2 sin ⇔ –2 = a cos 0
4
⇔ –2 = a
= 4 + 2 × (–1)
Grafik membentuk satu periode dari 0 sampai
=4–2
dengan 2π, periodenya adalah 2π sehingga nilai
=2

Diperoleh p = 2. k= 2π
= 1.
Nilai p – q = 2 – (–2) = 4. Jadi, nilai a = –2 dan k = 1.
Jadi, nilai p – q = 4.
29. Jawaban: c
26. Jawaban: c Misalkan grafik tersebut merupakan fungsi sinus
f(x) = a cos b(x + c) + d dengan bentuk umum y = a sin kx. Oleh karena nilai
Nilai f maksimum adalah fmaks = |a| + d dan nilai maksimumnya 2 dan nilai minimumnya –2 maka
minimum f adalah fmin = –|a| + d. π
nilai a = 2. Grafik membentuk satu periode dari – 2
π
f(x) = –5 cos (x – 4 ) + 1 3π
sampai dengan 2
, periodenya adalah 2π sehingga
f maks = |–5 |+ 1

=5+1=6 nilai k = 2π
= 1. Fungsi sementara adalah y = 2 sin x.
fmin = –|–5 |+ 1 π
Oleh karena grafik bergeser ke kiri sejauh 2
maka
= –5 + 1
π
= –4 fungsinya menjadi y = 2 sin (x + ). 2
Dengan demikian nilai f(x) terletak di antara –4
Jadi, fungsi yang sesuai dengan grafik tersebut
dan 6.
π
π adalah y = 2 sin (x + 2
).
Jadi, daerah hasil fungsi f(x) = –5 cos (x – 4
)+ 1
adalah {f(x) | –4 ≤ f(x) ≤ 6}. 30. Jawaban: a
f(x) = a cos b(x + c) + d dengan a = amplitudo dan
27. Jawaban: d b = periode.
Y
y = cos x y = cos (x –
π
) Nilai maksimum dan nilai minimum f(x) adalah
3
1 fmaks = 2 dan fmin = –2.
1 1
Amplitudo = 2 (fmaks – fmin) = 2 (2 – (–2)) = 2.
−π π• 5π• 3π 11π 5π 17π
X
6 2 6 2 6 2 6 Dengan demikian, a = 2.
–1
Grafik fungsi tepat mempunyai 1 periode pada
π
Grafik putus-putus merupakan grafik y1 = cos x. interval 3 sampai dengan π. Periode grafik =
π π 2π 2π
Titik ( 2 , 0) pada grafik y1 = cos x bergeser ke π – 3 = 3 . Dengan demikian, b = 2 π = 3.
5π π 2π π 3
kanan sejauh 6 – 2 = 6 = 3 satuan ke kanan. Persamaaan grafik fungsi sementara adalah
f(x) = 2 cos 3(x + c) + d.

Matematika Kelas X 45
Grafik fungsi tersebut merupakan grafik fungsi f(x) cos 2 x + sin2 x − 1
π =
= 2 cos 3x yang digeser ke kanan sejauh satuan (1 + sin x) cos x
3
1− 1
searah sumbu X. Persamaan grafiknya f(x) =
(1 + sin x) cos x
π
= 2 cos 3(x – 3 ) + d. = 0 = ruas kanan (terbukti)
Grafik melalui titik (0, –2), diperoleh:
(1 − cotan2 x) sin x
π b. Membuktikan = 1 + cotan x.
f(x) = 2 cos 3(x – 3)+d sin x − cos x

π Ruas kiri:
⇔ –2 = 2 cos (– 3 ) + d
(1 − cotan2 x) sin x
⇔ –2 = 2 × (–1) + d sin x − cos x
⇔ –2 = –2 + d
cos2 x
(1 − ) sin x
⇔ d=0 sin2 x
=
sin x − cos x
π
Persamaan grafiknya y = 2 cos 3(x – 3 )
sin2 x − cos2 x
⇔ y = 2 cos (π – 3x) ⇔ y = –2 cos 3x. ( ) sin x
sin2 x
Jadi, persamaan grafik fungsinya adalah y = –2 =
sin x − cos x
cos 3x.
sin2 x − cos 2 x
=
sin x (sin x − cos x)
B. Uraian
(sin x + cos x)(sin x − cos x)
1. a. Membuktikan tan A cos4 A + cotan A sin4 =
sin x (sin x − cos x)
A = sin A cos A. sin x + cos x
Ruas kiri: =
sin x
tan A cos4 A + cotan A sin4 A
cos x
sin A cos A
=1+
sin x
= × cos4 A + × sin4 A
cos A sin A = 1 + cotan x = ruas kanan(terbukti)
= sin A cos3 A + cos A sin3 A
= sin A cos A (cos2 A + sin2 A) AB2 + AC2 − BC2
3. cos BAC =
= sin A cos A × 1 2 × AB × AC
= sin A cos A = ruas kanan (terbukti)
312 + 25 2 − (24 2 )2
sin2 x cos2 x
=
2 × 31 × 25
b. Membuktikan –
cos2 x sin2 x
961 + 625 − 1.152
= sec2 x – cosec2 x. =
2 × 31 × 25
Ruas kiri: 434
= 2 × 31 × 25
sin2 x cos2 x
– = tan2 x – cotan2 x
cos2 x sin2 x 14 7
= 2 × 25
= 25
= (sec2 x – 1) – (cosec2 x – 1)
= sec2 x – cosec2 x 7
cos BAC = 25 maka sin BAC = 25
24
= ruas kanan (terbukti)
BC AC
cos x 1 + sin x
= sin ABC
sin BAC
2. a. Membuktikan – = 0.
1 + sin x cos x
24 2 24
Ruas kiri: ⇔ 24
= sin ABC
25
cos x 1 + sin x

1 + sin x cos x 24 2 24 12
⇔ sin ABC = 25 2
× = 50 2 =
25
2
cos 2 x (1 + sin x)(1 − sin x) 2
= –
(1 + sin x) cos x (1 + sin x) cos x 24 12
Jadi, sin BAC = 50 dan sin ABC = 25 2.
cos 2 x 1 − sin2 x
= –
(1 + sin x) cos x (1 + sin x) cos x

46 Trigonometri (2)
4. D C E Luas segitiga PQR
60°
1
= 2 × PQ × PR × sin P
1 63
60°
= 2 × 20 × 13 ×
30° 65
A 60 m B = 126 cm2
Jadi, luas segitiga PQR = 126 cm2.
AC AB
sin B
= 6. T
sin C
1
AB × sin B 60 × 2
⇔ AC = = = 30 m
sin C 1
AD
sin ∠ACD = AC C
A

⇔ AD = AC sin ∠ACD
18 cm
= 30 × sin 60°
B
1
= 30 × 3 Perhatikan ΔABC A C
2
∠A = ∠B = ∠C
= 15 3 ⇔ ∠A + ∠B + ∠C = 180°
Jadi, lebar sungai 15 3 m. ⇔ 3∠A = 180°
⇔ ∠A = 60°
5. Segitiga PQR beserta ukurannya disajikan seperti B
berikut. Luas segitiga ABC:
P 1
LABC = 2 × AB × AC × sin ∠A
1
20 cm 13 cm = 2 × 18 × 18 × sin 60°
1 1
= 2 × 18 × 18 × 2 3
Q R
21 cm
= 81 3 cm
a. Pada segitiga PQR berlaku aturan kosinus.
QR2 = PQ2 + PR2 – 2 × PQ × PR × cos P Luas Luas bidang empat T.ABC:
⇔ 212 = 202 + 132 – 2 × 20 × 13 × cos P LT.ABC = 4 × LABC
⇔ 441 = 400 + 169 – 520 cos P = 4 × 81 3
⇔ 441 = 569 – 520 cos P
⇔ 520 cos P = 569 – 441 = 324 3 cm
⇔ 520 cos P = 128 Jadi, luas bangun T.ABC = 324 3 cm2.
128
⇔ cos P = 7. Oleh karena panjang AC = BC maka segitiga ABC
520
16
merupakan segitiga sama kaki. Garis AC sejajar
⇔ cos P = dengan DE sehingga bentuk segitiga ABC beserta
65
16
ukurannya disajikan seperti berikut.
Jadi, nilai cos P = . C
65
16 4 cm
b. Telah diperoleh cos P = 65 , sehingga: 6 cm
E
2 cm
a = 652 − 162 A
30° 30° 30°
B
D
= 4.225 − 256 Besar ∠C, yaitu:
65
a ∠A + ∠B + ∠C = 180°
= 3.969
⇔ 30° + 30° + ∠C = 180°
= 63 ⇔ 60° + ∠C = 180°
P 16
63 ⇔ ∠C = 180° – 60°
sin P =
65 ⇔ ∠C = 120°
Besar ∠BED = ∠C = 120°.

Matematika Kelas X 47
Luas ADEC merupakan selisih antara luas ABC 1
dan luas DBE. Dengan demikian: Grafik fungsi mempunyai bentuk 2 periode
Luas ADEC
pada interval 0 sampai dengan 3π . Periode
= LABC – LDEB 4

1 grafik = 2 × ( 3π – 0) = 3π
.
= 2 × AC × BC × sin C 4 2
Grafik fungsi memotong sumbu Y di titik
1 (0, 0).
– 2
× DE × BE × sin ∠BEC Jadi, grafik fungsi f(x) mempunyai amplitudo 2,
1
= 2 × 6 × 6 × sin 120° – 2 × 2 × 2 × sin 120°
1 periode 3π , dan titik potong dengan sumbu
2

1 1 1 1
koordinat adalah (0, 0), ( 3π , 0), . . . .
2
= 2 ×6×6× 2 3 – 2 ×2×2× 2 3
π
9. a. f(x) = 2 sin (3x – 2 ) – 1
=9 3 – 3 =8 3
Menggunakan tabel nilai
Jadi, luas segi empat ADEC = 8 3 cm2. π
f(x) = 2 sin 3(x – 2 ) – 1.
8. a. Nilai maksimum dan nilai minimum f(x)
adalah fmaks = 4 dan fmin = 2. X f(x) (x, f(x))
1 1 0 –3 (0, –3)
Amplitudo = 2
( fmaks – fmin) = 2
(4 – 2) = 1.
2π 2π
9
0 ( 9
, 0)
Grafik fungsi tepat berulang pada interval –
π π
π π 3
1 ( 3 , 1)
6
sampai dengan 2
.
4π 4π
π 0 ( , 0)
Periode grafik = 3π – (– 6 ) = 4π = 2π .
9 9
6 6 3 2π 2π
3
–3 ( 3
, –3)
Grafik fungsi memotong sumbu Y di titik

(0, 3) dan tidak memotong sumbu X. 1 (

, 1)
9 9
Jadi, grafik fungsi f(x) mempunyai amplitudo 1,
10π 10π
0 ( , 0)
2π 9 9
periode , dan titik potong dengan sumbu
3 14π 14π
9
0 ( , 0)
koordinat adalah (0, 3). 9

5π 5π
b. Nilai maksimum dan nilai minimum g(x) 3
1 ( 3
, 1)
adalah gmaks = 2 dan gmin = –2.
2π –3 (2π, –3)
1 1
Amplitudo = 2 ( gmaks – gmin) = 2 ( 2 – (–2)) = 2. π
Grafik fungsi f(x)= 2 sin (3x – 2 ) – 1.
Grafik fungsi tepat berulang pada interval
0 sampai dengan π. Periode grafik = π – 0 = π. Y π
f(x) = 2 sin (3x – )–1
Grafik fungsi memotong sumbu Y di titik 1 2

π
(0, –2) dan memotong sumbu X di ( 2 , 0), X
0 2π 8π 10π 14π 16π
4π 2π
9 9
( 3π
9 9 9 9
, 0). –1
4
Jadi, grafik fungsi g(x) mempunyai amplitudo
2, periode π, dan titik potong dengan sumbu –2
π
koordinat adalah (0, –2), ( 4
, 0), ( 3π , 0), –3
4
( 5π , 0), ( 7π , 0).
4 4
c. Nilai maksimum dan nilai minimum h(x)
adalah hmaks =0 dan hmin = –4.
1 1
Amplitudo = 2 ( hmaks – hmin) = 2 ( 0 – (–4)) = 2.

48 Trigonometri (2)
π Titik (0, 2) pada grafik y = 2 cos x bergeser 0
b. g(x) = 4 – cos 3(x – 2 )
satuan searah sumbu x dan 1 satuan ke atas
Menggunakan tabel nilai searah sumbu Y (yaitu menjadi titik (0, 3)).
π
g(x) = 4 – cos 3(x – 2 ). Dengan demikian c = 0 dan d = 1.
Persamaan grafiknya:
X g(x) (x, g(x) f(x) = 2 cos 3(x + 0) + 2 = 2 cos 3x + 2.
0 4 (0, 4)
Jadi, persamaan grafiknya f(x) = 2 cos 3x + 2.
π π Cara lain:
5 ( 6 , 5)
6 Fungsi f(x) dapat dipandang sebagai fungsi
π
4
π
( 3 , 4)
sinus.
3
f(x) = a sin b(x + c) + d dengan a = amplitudo
π
3
π
( 2 , 3) dan b = periode
2
Nilai maksimum dan nilai minimum f(x)
2π 2π
3
4 ( 3
, 4) adalah fmaks = 3 dan fmin = –1.
5π 5π
1 1
6
5 ( , 5) Amplitudo = 2 (fmaks – fmin) = 2 (3 – (–1)) = 2.
6
Dengan demikian, a = 2.
π 4 (π, 4)
Grafik fungsi tepat mempunyai 1 puncak 1

lembah pada interval 0 sampai dengan 2π .

6
3 ( 6
, 3)
3
4π 4π
4 ( , 4) 2π
3 3 b= 2π
=3
3π 5 3π 3
2
( 2
, 5)
Persamaan fungsi sementara = 2 sin
2π 4 (π, 4) 3 (x + c) + d
π
Titik (0, 0) pada grafik fungsi y = sin 2x ber-
Grafik fungsi g(x) = 4 – cos 3(x – 2 ) π
geser ke kiri sejauh 6 satuan searah sumbu X
Y π
g(x) = 4 – cos 3(x – 2
) dan ke atas sejauh 1 satuan searah sumbu Y.
5
π
Dengan demikian, c = 6 dan d = 1.
4
Persamaan grafiknya:
3 π
f(x) = 2 sin 3(x + 6 ) + 1.
2
Jadi, persamaan grafiknya f(x) = 2 sin
1 π
3(x + 6 ) + 1.
X
π π π 2π 5π π 7π 4π 3π 2π b. Misalkan g(x) = a cos b(x + c) + d dengan a
6 3 2 3 6 6 3 2
= amplitudo dan b = periode.
10. a. Misalkan f(x) = a cos b(x + c) + d dengan a
Nilai maksimum dan nilai minimum f(x)
= amplitudo dan b = periode
adalah gmaks = 0 dan gmin = –2.
Nilai maksimum dan nilai minimum f(x)
1 1
adalah fmaks = 3 dan fmin = –1. Amplitudo = 2 (gmaks – gmin) = 2 (0 – (–2)) = 1.
1
Amplitudo = 2 (fmaks – fmin) = (3 – (–1)) = 2. Dengan demikian, a = 1.
Grafik fungsi tepat mempunyai 1 periode π
Dengan demikian, a = 2. lembah pada interval 0 sampai dengan π.
Grafik fungsi tepat mempunyai 1 periode 2π

b= π =2
pada interval 0 sampai dengan sehingga
3 Persamaan fungsi sementara:
periodenya 2π . g(x) = cos 2(x + c) + d
3 Titik (0, 1) pada grafik y = cos 2x bergeser ke
2π 2π π
b= = 2π = 3. kanan sejauh 2 satuan dan bergeser ke
periode 3
bawah sejauh 1 satuan. Dengan demikian
Persamaan fungsi sementara:
π
f(x) = 2 cos (x + c) + d c = – 2 dan d = –1.

Matematika Kelas X 49
Persamaan grafiknya: Grafik fungsi tepat mempunyai 1 periode 1
π pada interval 0 sampai dengan π sehingga
g(x) = cos 2(x – ) – 1 = cos –(π – 2x) – 1 =
2 periode grafik adalah π. Dengan demikian b
– cos 2x – 1.

Jadi, persamaan grafiknya g(x) = –cos 2x – 1. = π = 2.
Cara lain Persamaan fungsi sementara:
Fungsi f(x) dapat dipandang sebagai fungsi g(x) = sin 2(x + c) + d
sinus. Titik (0, 0) pada grafik y = sin 2x bergeser ke
g(x) = a sin b(x + c) + d dengan a = amplitudo π
dan b = periode. kanan sejauh 4 satuan dan bergeser ke
Nilai maksimum dan nilai minimum f(x) bawah sejauh 1 satuan. Dengan demikian,
adalah gmaks = 0 dan gmin = –2. π
c = 4 dan d = –1.
1 1
Amplitudo = (g
2 maks
– gmin) = 2 (0 – (–2)) = 1. Persamaan grafiknya:
Dengan demikian, a = 1. π
y = sin 2(x – 4 ) – 1
π
Jadi, persamaan grafiknya f(x) = sin 2(x – 4 ) – 4.

50 Trigonometri (2)
A. Pilihan Ganda (iv) Sudut D = –95° sama dengan sudut 265°.
1. Jawaban: d Nilainya di antara 180° dan 270° sehingga
terletak di kuadran III. Pernyataan (iv) benar.
180°
1 rad = π
sehingga: Jadi, pernyataan yang benar adalah (i), (ii), dan
5 5 180°
(iv).
6
π rad = 6 π × π = 150°
4. Jawaban: c
5 24°46'38'' + 12°21'54''
Jadi, sudut 6 π rad = 150°.
= (24° + 12°) + (46' + 21') + (38'' + 54'')
2. Jawaban: a = 36° + 67' + 92''
Sudut selalu dihitung dari sumbu X positif yang = 36° + 60' + 7' + 60'' + 32''
diputar berlawanan arah putaran jarum jam dengan = 36° + 1° + 7' + 1' + 32''
titik asal (titik O) sebagai pusat putaran. Dengan = 37° + 8' + 32''
demikian sudut 45° disajikan seperti berikut. = 37°8'32''
Y Jadi, hasil 24°46'38'' + 12°21'54'' = 37°8'32''.
5. Jawaban: e
45°54'21'' – 23°43'51'' + 2°4'12''
45°
X
= (45° – 23° + 2°) + (54' – 43' + 4')
O + (21'' – 51'' + 12'')
= 24° + 15' + (21'' – 51'' + 12'')
= 24° + 14' + 1' + (21'' – 51'' + 12'')
= 24° + 14' + (60'' + 21'' – 51'' + 12'')
Jadi, sudut 45° ditunjukkan pada pilihan a. = 24° + 14' + 42''
= 24°14'42''
3. Jawaban: c Jadi, 45°54'21'' – 23°43'51'' + 2°4'12'' = 24°14'42''.
Sudut di beberapa kuadran sebagai berikut.
1) Kuadran I terletak di antara 0° dan 90°. 6. Jawaban: b
2) Kuadran II terletak di antara 90° dan 180°. Segitiga KLM siku-siku di L dengan α dan β masing-
3) Kuadran III terletak di antara 180° dan 270°. masing terletak pada sudut M dan sudut K.
4) Kuadran IV terletak di antara 270° dan 360°. Perbandingan trigonometri pada segitiga tersebut
sebagai berikut.
Pada pernyataan tersebut:
sisi depan sudut α KL
(i) Sudut A = 105° nilainya di antara 90° dan (i) sin α = =
sisi miring sudut α KM
180° sehingga terletak di kuadran II. sisi depan sudut β LM
Pernyataan (i) benar. (ii) tan β = =
sisi samping sudut β KL
(ii) Sudut B = 185° nilainya di antara 180° dan sisi miring sudut α KM
(iii) sec α = =
270° sehingga terletak di kuadran III. sisi samping sudut α LM
Pernyataan (ii) benar. sisi miring sudut β KM
(iv) cosec β = sisi depan sudut β = LM
(iii) Sudut C = –15° sama dengan sudut 345°.
sisi miring sudut β KM
Nilainya di antara 270° dan 360° sehingga (v) sec β = sisi samping sudut β = KL
terletak di kuadran IV. Pernyataan (iii) salah.
Jadi, perbandingan trigonometri yang benar
ditunjukkan oleh (i), (ii), dan (v).

Matematika Kelas X 51
7. Jawaban: c (ii) ∆BCD siku-siku di D
Segitiga ABC disajikan seperti berikut.
Y
CD = BC2 – BD2
3
2
= 172 – 82
C 1 B = 289 – 64
X
–3 –2 –1 0 1 2 3 = 225 = 15 cm
–1
–2 Nilai cotan θ diperoleh dari perbandingan antara
–3 sisi samping sudut θ dengan sisi depan sudut θ.
–4 CD 15
A cotan θ = BD = 8
Segitiga ABC siku-siku di B dengan panjang AB 15
Jadi, nilai cotan θ = 8 .
= 5 satuan dan BC = 4 satuan.
Panjang AC: 10. Jawaban: d
Nilai sin B diperoleh dari perbandingan antara sisi
AC = AB2 + BC2 AC dengan sisi BC. Dengan demikian:
= 52 + 42 = 25 + 16 = 41 1 AC 1
sin B = 3 ⇔ BC = 3
sisi depan sudut A
sin A = Misalkan panjang AC = a satuan maka panjang
sisi miring sudut A
BC 4 4 BC = 3a satuan.
= AC = 41 = 41 41 ∆ABC siku-siku di A, berlaku:
4 AB2 + AC2 = BC2
Jadi, nilai sin A = 41 41 .
⇔ (6 2 )2 + a2 = (3a)2
8. Jawaban: e ⇔ 72 + a2 = 9a2
Segitiga PQR siku-siku di R sehingga nilai cos P ⇔ 72 = 8a2
diperoleh dari perbandingan antara sisi PR ⇔ a2 = 9
dengan sisi PQ. ⇔ a=3
Panjang PR: Jadi, panjang AC = a = 3 cm.
PR = PQ 2 – QR 2 11. Jawaban: d
∆KLM disajikan seperti berikut.
= (3 13)2 – 92
M
= 117 – 81
= 36 = 6 4
3
PR 6 2
cos P = PQ = 3 13 = 13 13
2 K L
Jadi, nilai cos P = 13 13 .
Nilai sinus diperoleh dari perbandingan antara sisi
9. Jawaban: a depan sudut dengan sisi miring sudut. Oleh
Gambar tersebut disajikan kembali seperti berikut. 3
karena nilai sin L = 4 sehingga:
(i) ∆ABD siku-siku di B
A 3 KM 3
BD = AD2 – AB2 sin L = 4 ⇔ LM = 4
10 cm
6 cm
Misalkan panjang KM = 3 satuan maka panjang
= 102 – 62 B D
LM = 4 satuan.
= 100 – 36 Panjang KL:

= 64 17 cm KL = LM2 – KM2
= 8 cm
= 42 – 32
θ
= 7
C

52 Ulangan Akhir Semester


KM 3 3 Panjang AB:
cotan M = KL = 7
= 7 7
AB = AC2 – BC2
3
Jadi, nilai cotan M = 7.
7 = (3 5)2 – (2 5)2
12. Jawaban: b
= 45 – 20
∆PQR disajikan seperti berikut.
P = 25
= 5 cm
1 Jadi, panjang sisi AB = 5 cm.
14. Jawaban: a
Gambar tersebut dapat disajikan ulang seperti
Q m R
berikut.
Nilai cos R diperoleh dari perbandingan antara Y
sisi QR dengan sisi PR. Oleh karena cos R = m A
sehingga:
QR 4
cos R = m ⇔ PR = m
β θ
Misalkan panjang QR = m satuan maka panjang X
B 3 O
PR = 1 satuan.
Panjang PQ:
Panjang AO:
PQ = PR 2 – QR 2 = 1 – m2
QR m
AO = AB2 + BO 2
tan P = PQ =
1 – m2 = 42 + 32
m =
Jadi, nilai tan P = . 16 + 9
1 – m2
= 25 = 5
13. Jawaban: d Misalkan ∠BOA = β diperoleh:
∆ABC disajikan seperti berikut. AO 5
sec β = BO = 3
A
Sudut θ dan β saling berpelurus sehingga
θ = 180° – β.
5
sec θ = sec (180° – β) = –sec β = – 3
B C 5
20 cm Jadi, nilai sec θ = – 3 .
Nilai kosinus diperoleh dari perbandingan antara
15. Jawaban: c
sisi samping sudut dengan sisi miring sudut. Oleh
2 (i) ∆ADC siku-siku di D
karena cos C = 3 maka: CD
tan ∠CAD = AD
2 BC 2
cos C = 3 ⇔ AC
= 3 CD
⇔ tan 30° = 2 6
20 2 CD
⇔ = 3 ⇔
3
= 2 6
AC
3
2 5 2 ⇔ 3CD = 6 2
⇔ = 3
AC
⇔ CD = 2 2 cm
⇔ AC = 3 5 cm

Matematika Kelas X 53
(ii) ∆BCD siku-siku di D 3+2 3 +1
CD =
3–1
sin ∠DBC = BC
4+2 3
⇔ sin 45° =
2 2 =
2
BC
2 2 2 =2+ 3
⇔ =
2 BC
sin 120° + sin 150°
⇔ BC = 4 cm Jadi, nilai cos 330° + cos 240° = 2 + 3.
Jadi, panjang BC = 4 cm.
19. Jawaban: a
16. Jawaban: c Nilai tangen diperoleh dari perbandingan antara
2 2 sisi depan sudut dengan sisi samping sudut. Nilai
3
π = 3 × 180° = 120° sehingga:
2
tan θ = 5 5 dapat dibentuk segitiga seperti
cosec 120° = cosec (180° – 60°)
berikut.
= cosec 60°
1
= sin 60° r
2 5
1 2 2
= 1
= 3
= 3 3
3 θ
2
5
2 2
Jadi, nilai cosec 3
π= 3
3. Nilai r:
17. Jawaban: d r= 52 + (2 5)2
π π 5π
tan2 6 – sec2 4 + cosec2 6 = 25 + 20
= tan2 30° – sec2 45° + cosec2 150° = 45 = 3 5
= tan2 30° – sec2 45° + cosec2 (180° – 30°) π 5 1
sin (2 – θ) = cos θ = 3 5 = 3 5
= tan2 30° – sec2 45° + cosec2 30°
π 1
3 2 2 Jadi, nilai sin ( 2 – θ) = 3 5 .
=( ) – ( 2 )2 + (2)2
3
20. Jawaban: e
1
= 3 –2+4 Nilai sinus diperoleh dari perbandingan antara sisi
1 depan sudut dengan sisi miring sudut. Nilai sin α
=23 4
= 5 dapat dibentuk segitiga seperti berikut.
π π 5π 1
Jadi, nilai tan2 6 – sec2 4 + cosec2 6 = 23 .
5
18. Jawaban: b 4
sin 120° + sin 150°
α
cos 330° + cos 240°
p
sin (180° – 60°) + sin (180° – 30°)
= cos (360° – 30°) + cos (180° + 60°) Nilai p:

sin 60° + sin 30° p= 52 – 42


= cos 30° – cos 60°
= 9 =3
1
3+
1 tan (180° + α) – sin (270° – α)
2 2
= 1 1 = tan α – (–cos α)
3–
2 2 = tan α + cos α
3 +1 3 +1 4 3 29 14
= × = 3 + 5 = 15 = 1 15
3 –1 3 +1
14
Jadi, nilai tan (180° + α) – sin (270° – α) = 1 15 .

54 Ulangan Akhir Semester


21. Jawaban: b 25. Jawaban: a
2 sin2 α × cotan α – (cos α + sin α)2 ∠MKL = 180° – ∠KLM – ∠LMK
cos α = 180° – 80° – 55° = 45°
= 2 × sin2 α × sin α – (cos2 α + 2 cos α sin α + sin2 α)
Luas segitiga (s, sd, s):
= 2 sin α cos α – 2 sin α cos α – (cos2 α + sin2 α)
1
= –(cos2 α + sin2 α) = –1 L∆KLM = 2 × KL × KM × sin ∠MKL
Jadi, hasil dari 2 sin 2 α × cotan α – (cos α 1
= 2 × 15 × 18 × sin 45°
+ sin α)2 = –1.
1 1
22. Jawaban: c = 2 × 15 × 18 × 2 2
5sin2 α − cos2 α + 1 5 sin2 α + (1 − cos2 α) 135
= = 2 2 cm2
2 cosec α 2× 1
sin α
135
5 sin2 α + sin2 α Jadi, luas segitiga KLM = 2 2 cm2.
= 2
sin α
26. Jawaban: b
6 sin2 α
= 2
D
sin α
2x
= 3 sin2 α × sin α 2 cm
= 3 sin3 α 60° C
2 2 x
5 sin α − cos α + 1 60°
Jadi, bentuk sederhana adalah
2 cosec α A 3 cm B
3 sin3 α.
Misalkan panjang BC = x, maka DC = 2x.
23. Jawaban: a Perhatikan ∆ABD.
Menggunakan aturan kosinus: BD2 = AB2 + AD2 – 2 × AB × AD × cos BAD
BC2 = AB2 + AC2 – 2 × AB × AC × cos A = 32 + 22 – 2 × 3 × 2 × cos 60°
= 162 + 102 – 2 × 16 × 10 × cos 60° 1
= 256 + 100 – 160 = 9 + 4 – 12 × 2
C
= 356 – 160 = 13 – 6 = 7
= 196 10 cm BD = 7 cm
BC = 196 = 14 60° Perhatikan ∆BCD.
Jadi, panjang BC = 14 cm. A 16 cm B BD2 = BC2 + CD2 – 2 × BC × CD × cos BCD
⇔ 7 = x2 + (2x)2 – 2 × x × 2x × cos 60°
24. Jawaban: a 1
C ⇔ 7 = x2 + 4x2 – 4x2 × 2
⇔ 7 = 5x2 – 2x2
16 cm 16 2 cm ⇔ 7 = 3x2
7
⇔ x2 = 3
A 30° 7 3 21 1
B ⇔ x=± × = = ± 3 21
3 3 9
Pada segitiga ABC berlaku aturan sinus, yaitu: Oleh karena x merupakan satuan panjang maka
AC BC 1
= x > 0 sehingga x = 3 21 .
sin ∠ABC sin ∠BAC
16 16 2 1
⇔ = Jadi, panjang BC = x = 3 21 cm.
sin 30° sin ∠BAC
16 16 2 27. Jawaban: b
⇔ =
1 sin ∠BAC Misalkan ∠CAB = 19x, ∠ABC = 60x, ∠BCA = 11x.
2

⇔ sin ∠BAC =
16 2 ∠CAB + ∠ABC + ∠BCA = 180°
32
⇔ 19x + 60x + 11x = 180°
1
⇔ sin ∠BAC = 2 2 ⇔ 90x = 180°
1
⇔ x = 2°
sin ∠BAC = 2 2 ⇔ ∠BAC = 45°
Jadi, tan ∠BAC = tan 45° = 1.

Matematika Kelas X 55
∠ABC = 60x = 120° 30. Jawaban: a
Luas segitiga (s, sd, s): Perhatikan bahwa ∆ABC merupakan segitiga
1 sama kaki.
L∆ABC = 2 × AB × BC × sin ∠B ∠ABC = 180° – 2 × ∠BAC
1 = 180° – 60°
= 2 × 12 × 20 × sin 120° = 120°
1 Luas segitiga ABC:
= 120 × 2 3
1
L∆ABC = 2 × AB × BC × sin ∠ABC
= 60 3 cm2
1
Jadi, luas ∆ABC = 60 3 cm2. = 2 × 30 × 30 × sin 120°
1 1
28. Jawaban: c B = 2 × 900 × 3
2
Segi enam beraturan = 225 3 cm2
terdiri atas 6 segitiga sama Volume prisma:
60° A
sisi yang kongruen.
O Vprisma = L∆ABC × CF
Besar sudut pusat
360° = 225 3 × 40
= 6
= 60°
= 9.000 3 cm3
Luas segi enam beraturan = 6 × L∆AOB Jadi, volume prisma tersebut 9.000 3 cm3.
1 31. Jawaban: b
⇔ 216 3 = 6 × 2 × OA × OB × sin AOB
BC
⇔ 216 3 = 3 × AB × AB × sin 60° BE : BC = 1 : 2 ⇔ BE = 2
1 CA = BC = 4 cm sehingga ∆ABC merupakan
⇔ 216 3 = 3AB × AB × 2 3
segitiga sama kaki (∠CAB = ∠CBA = ∠EBD = 30°).
1 AC // DE sehingga ∠EDB = ∠CAB = 30°.
⇔ 216 = 3AB2 × 2
∠BCA = ∠BED = 180° – 2 × 30° = 120°
2
⇔ AB2 = 216 × 3 Misalkan luas segi empat ADEC = L
L = L∆ABC – L∆DBE
⇔ AB2 = 144
1 1
⇔ AB = ± 144 = 12 cm = 2 × AC × BC × sin ∠BED – 2 × BE × DE × sin ∠BED
Oleh karena AB merupakan panjang sisi maka 1 1
= 2 × BC × BC × sin 120° – 2 × BE × BE × sin 120°
AB > 0 yaitu AB = 12 cm.
1
Jadi, panjang sisi segi enam beraturan tersebut = 2 × sin 120° × (BC2 – BE2)
12 cm.
1 BC
29. Jawaban: c = × 1 3 (BC2 – ( 2 )2)
2 2
Perhatikan ∆ODA dan ∆ODC.
1
∠ODA = ∠ODC = 3 (42 – 22)
4
= 180° – ∠AOD – ∠DAO
= 1 3 × 12
= 180° – 90° – 60° 4
= 30° = 3 3 cm2
Luas bangun datar ABCD: Jadi, luas segi empat ADEC = 3 3 cm2.
L = 2 × luas segitiga ABD
1
32. Jawaban: d
= 2 × 2 × AD × BD × sin ∠BDA Nilai maksimum sin x untuk 0 ≤ x ≤ 2π adalah 1.
1 Nilai minimum sin x untuk 0 ≤ x ≤ 2π adalah –1.
= 2 × 2 × 20 × 30 × sin 30°
fmaks = 1 + 1 = 2
1 fmin = –1 + 1 = 0
= 600 × 2
Jadi, daerah hasilnya f(x) = sin x + 1 adalah
= 300 cm2 {f(x) | 0 ≤ f(x) ≤ 2, f(x) ∈ R}.
Jadi, luas bangun datar ABCD adalah 300 cm2.

56 Ulangan Akhir Semester


33. Jawaban: d a = nilai fmaks = 5
Grafik y = 2 sin 2(x + 30°) dapat digambar dengan b = nilai fmin = –3
membuat sketsa grafik fungsi y1 = sin 2x terlebih
(a – b)2 = (5 – (–3))2 = 82 = 64
dahulu kemudian menggeser grafik fungsi
y1 = sin 2x ke kiri sejauh 30° sehingga diperoleh Jadi, nilai (a – b)2 = 64.
grafik y 2 = sin 2(x + 30°). Setiap ordinat 36. Jawaban: e
y2 = sin 2(x + 30°) dikalikan dengan dua kemudian Grafik fungsi g(x) = cos (x – 45°) memotong
dibuat titik-titik baru yang dilalui grafik sumbu X jika y = g(x) = 0.
y3 = 2 sin 2(x + 30°). Grafik y3 = 2 sin 2(x + 30°) g(x) = 0
diperoleh dengan menghubungkan titik-titik ⇔ cos (x – 45°) = 0
tersebut. ⇔ cos (x – 45°) = cos (±90°)
Y ⇔ x – 45° = (±90°) + k × 360°
2
⇔ x = 45° ± 90° + k × 360°
1 ⇔ x = 135° + k × 360° atau x = –45° + k × 360°
X 1) x = 135° + k × 360°
–30° 0 60° 90° 150° 240° 330° 360°
Untuk k = 0 diperoleh x = 135°.
–1
Untuk k = 1 diperoleh x = 495°.
–2
2) x = –45° + k × 360°
Jadi, grafik yang sesuai pada pilihan d. Untuk k = 0 diperoleh x = –45°
Untuk k = 1 diperoleh x = 315°.
34. Jawaban: c
π Jadi, grafik fungsi g(x) = cos (x – 45°) memotong
Grafik fungsi y = 1 – cos (x – 3 ) dapat diperoleh sumbu X di titik (135°, 0) dan (315°, 0).
dengan menggambar y1 = cos x terlebih dahulu, 37. Jawaban: d
kemudian menggesernya ke kanan untuk Nilai fmaks = 3 dan nilai fmin = –1.
π
mendapatkan grafik fungsi y2 = cos (x – 3 ). Untuk 1
Amplitudo = 2 (3 – (–1)) = 2.
π
mendapatkan grafik fungsi y3 = –cos (x – 3
), Grafik fungsi mempunyai bentuk tepat 1 puncak
π π 5π
setiap ordinat grafik fungsi y2 = cos (x – 3
) dan 1 lembah pada interval 4 sampai dengan 4 .
dikalikan dengan (–1). Dengan menggeser grafik 5π π
π Periode grafik fungsi = 4 – 4 = π.
fungsi y3 = –cos (x – 3 ) ke atas 1 satuan searah
Jadi, amplitudo dan periode grafik fungsi f(x)
π
sumbu Y, diperoleh grafik fungsi y = 1 – cos (x – 3 ). berturut-turut adalah 2 dan π.
Pada gambar berikut, grafik fungsi y1 = cos x tidak 38. Jawaban: c
digambar. Misalkan persamaan grafik fungsi tersebut
Y y = a sin b(x – c) + d.
π
2 y = 1 – cos (x –
3
) Nilai ymaks = 4 dan nilai ymin = 0.
π
y2 = cos (x – )
1 3 1
Amplitudo = 2 (4 – 0) = 2. Dengan demikian, nilai
−π 0 π 2π π 4π 5π 2π
X a = amplitudo = 2.
3 3
–1
3 3 3
y3 = –cos (x –
π
3
) Grafik fungsi mempunyai bentuk tepat 1 puncak
π π 7π
Jadi, grafik fungsi y = 1 – cos (x – ) adalah dan 1 lembah pada interval 2 sampai dengan 6 .
3
pilihan c. 7π 3π 4π 2π
Periode grafik fungsi = 6 – 6 = 6 = 3 .
35. Jawaban: d
2π 2π 3
π Nilai b = = = 2.
Fungsi f(x) = –4 sin 3(x – 2 ) + 1 mempunyai nilai periode 2π
3

maksimum dan nilai minimum: Persamaan grafik fungsi sementara


3
fmaks = |–4| + 1 = 4 + 1 = 5 y = 2 sin 2 (x – c) + d.
fmin = –|–4| + 1 = –4 + 1 = –3

Matematika Kelas X 57
Titik potong dengan sumbu Y pada grafik 2π 2π 1
Nilai b = = = 2.
3 periode 4π
y = 2 sin 2
x, yaitu titik (0, 0), bergeser ke kanan
Persamaan grafik fungsi sementara y =
sejauh 0 satuan searah sumbu X dan bergeser 1
ke atas 2 satuan searah sumbu Y. Dengan sin 2 (x – c) + d.
demikian c = 0 dan d = 2. Titik potong dengan sumbu Y pada grafik y =
Jadi, persamaan grafik fungsi tersebut adalah
1
3 3 sin 2 x, yaitu titik (0, 0), bergeser ke kanan sejauh
y = 2 sin 2 (x – 0) + 2 = 2 sin ( 2 x) + 2.
11π
39. Jawaban: e satuan searah sumbu X dan bergeser ke atas
6
Misalkan persamaan grafik fungsi tersebut
1 satuan searah sumbu Y. Dengan demikian,
y = a cos b(x – c) + d.
11π
Nilai ymaks = 1 dan nilai ymin = –5. c= dan d = 1.
6
1
Amplitudo = (1 – (–5)) = 3. Dengan demikian, Persamaan grafik fungsi:
2
1 11π
nilai a = amplitudo = 3. y = sin 2 (x – )+1
6
Grafik fungsi mempunyai bentuk tepat 1 puncak 1 π
π 7π
= sin ( 2 x – π + )+1
dan 1 lembah pada interval sampai dengan . 12
2 2 1 π
7π π 6π = sin – (π – ( 2 x + )) + 1
Periode grafik fungsi = – = = 3π. 12
2 2 2 1 π
= –sin (π – ( 2 x + )) + 1
2π 2π 2 12
Nilai b = = = 3
. 1
periode 3π π
= –sin ( 2 x + )+1
Persamaan grafik fungsi sementara 12
2 1 π
y = 3 cos 3 (x – c) + d. = 1 – sin 2 (x + )
6
Titik potong dengan sumbu Y pada grafik Jadi, persamaan grafik fungsi y
2 1 π
y = 3 cos 3 x, yaitu titik (0, 3), bergeser ke kanan = 1 – sin 2 (x + ).
6
π
sejauh 2 satuan searah sumbu X dan bergeser B. Uraian
ke bawah 2 satuan searah sumbu Y. Dengan 1. a. 24°46'54'' + 67°34'11''
π = (24° + 67°) + (46' + 34') + (54'' + 11'')
demikian, c = 2
dan d = –2.
= 91° + 80' + 65''
2 π
Jadi, persamaan grafik fungsi y = 3 cos 3 (x – 2 ) = 91° + 60' + 20' + 60'' + 5''
2 π = 91° + 1° + 20' + 1' + 5''
+ (–2) = 3 cos 3 (x – 2 ) – 2. = 92° + 21' + 5''
40. Jawaban: c = 92°21'5''
Misalkan persamaan grafik fungsi tersebut Jadi, 24°46'54'' + 67°34'11'' = 92°21'5''.
y = a sin b(x – c) + d. b. 34°12'25'' – 18°45'12'' – 15°25'39''
Nilai ymaks = 2 dan nilai ymin = 0. = (34° – 18° – 15°) + (12' – 45' – 25')
1 + (25'' – 12'' – 39'')
Amplitudo = 2 (2 – 0) = 1.
= 1° + (12' – 45' – 25') + (25'' – 12'' – 39'')
Dengan demikian, nilai a = amplitudo = 1.
= (60' + 12' – 45' – 25') + (25'' – 12'' – 39'')
Grafik fungsi mempunyai bentuk tepat 1 puncak
= 2' + (25'' – 12'' – 39'')

dan 1 lembah pada interval – 6 sampai dengan = 1' + (1' + 25'' – 12'' – 39'')
17π = 1' + (60'' + 25'' – 12'' – 39'')
6
.
= 1' + 34''
17π 7π 24π
Periode grafik fungsi = 6 – (– 6 ) = 6 = 4π. = 1'34''
Jadi, 34°12'25'' – 18°45'12'' – 15°25'39'' = 1'34''.

58 Ulangan Akhir Semester


2. a. Segitiga tersebut dapat disajikan kembali 3 6
⇔ = LN
sebagai berikut. 2
12
∆ABC siku-siku di C. ⇔ LN = =4 3
3
2 2 C
BC = AB – AC Jadi, panjang LN = 4 3 cm.
8
= 172 – 82 b. Panjang KL
θ Pada ∆KLN berlaku perbandingan trigono-
= 289 – 64
A 17 B metri sebagai berikut.
= 225 = 15 KL
sin ∠LNK = LN
sisi depan sudut θ AC 8
(i) sin θ = sisi miring sudut θ = AB = 17 KL
⇔ sin 45° = 4 3
sisi samping sudut θ BC 15
(ii) cos θ = sisi miring sudut θ = AB = 17 2 KL
⇔ = 4 3
2
sisi depan sudut θ AC 8 4 3× 2
(iii) tan θ = sisi samping sudut θ = BC = 15 ⇔ KL = =2 6
2
b. Segitiga tersebut dapat disajikan kembali Jadi, panjang KL = 2 6 cm.
sebagai berikut.
D 4
4. Cermati gambar berikut.
C
θ D C
5 60°

30°
A
6 B x cm
∆BCD siku-siku di D. 60°
O
E 45°
2 2
BD = BC – CD (2 – x) cm
45°
= 52 – 42 A 2 cm B

= 25 – 16 Tarik garis tinggi dari titik E tegak lurus dengan


sisi BC sehingga berpotongan di titik O. Misalkan
= 9 =3 panjang OC = x cm maka panjang BO = (2 – x) cm.
∆ABD siku-siku di B. Segitiga BOE merupakan segitiga siku-siku sama
kaki sehingga panjang EO = BO = (2 – x) cm.
AD = AB2 + BD2 Pada ∆EOC berlaku perbandingan trigonometri
berikut.
= 62 + 32 EO
tan ∠OCE = OC
= 36 + 9
2–x
= 45 = 3 5 ⇔ tan 30° = x
AB 6 2 3 2–x
(i) sin θ = AD = 3 5 = 5 5 ⇔ = x
3

BD 3 1 ⇔ x 3 = 6 – 3x
(ii) cos θ = AD = 3 5 = 5 5
⇔ x(3 + 3)=6
AB 6 6
(iii) tan θ = BD = 3 = 2 ⇔ x= 3+ 3
6 3– 3
3. a. Panjang LN ⇔ x= 3+ 3 ×
3– 3
Pada ∆LMN berlaku perbandingan trigono-
6(3 – 3)
metri sebagai berikut. ⇔ x= =3– 3
6
LM
cos ∠NLM = LN
6
⇔ cos 30° = LN

Matematika Kelas X 59
1 sin x sin y + cos y cos x
Luas ∆BCE = 2 × BC × EO = cos x sin y
cos x cos y + sin y sin x
1 cos y sin x
= 2
× 2 × (2 – x)
sin x sin y + cos y cos x cos y sin x
= ×
=2–x cos x sin y cos x cos y + sin y sin x

= 2 – (3 – 3) cos y sin x
=
cos x sin y
= 3 –1 cos y
= sin x ×
cos x sin y
Jadi, luas segitiga BCE = ( 3 – 1) cm2.
= tan x cotan y (Dapat dibuktikan)
5. Diketahui α merupakan sudut lancip. Nilai tangen
diperoleh dari perbandingan antara sisi depan 7. a. R
sudut dengan sisi samping sudut. Nilai tan α = m 26 cm
dapat dibentuk segitiga seperti berikut. 16 cm
Q
r
m
12 cm
α
P
1
PQ2 = PR2 + QR2 – 2PR × QR × cos ∠QRP
Nilai r:
⇔ 122 = 162 + 262 – 2 × 16 × 26 × cos ∠QRP
r= m2 + 12 = m2 + 1 ⇔ 144 = 256 + 676 – 832 × cos ∠QRP
m ⇔ 144 = 932 – 832 × cos ∠QRP
Diperoleh sin α = .
m2 + 1 ⇔ 832 cos ∠QRP = 788
Dengan demikian: 788
⇔ cos ∠QRP =
sin α = cos β 832
⇔ sin α = sin (90° – β) 197
⇔ α = 90° – β =
208
⇔ α + β = 90° 197
Jadi, besar sudut (α + β) = 90°. Jadi, nilai cos ∠QRP = .
208

cos x 1 + sin x 1
6. a. + b. s = 2 (PQ + QR + PR)
1 + sin x cos x
cos x 1 − sin x 1 + sin x 1
= × + = 2 (12 + 26 + 16)
1 + sin x 1 − sin x cos x
cos x(1 − sin x) 1 + sin x
= + = 27
1 − sin2 x cos x
cos x(1 − sin x) 1 + sin x Luas segitiga PQR (s, s, s):
= +
cos 2 x cos x
L = s(s − PQ)(s − QR)(s − PR)
1 − sin x 1 + sin x
= +
cos x cos x = 27(27 − 12)(27 − 26)(27 − 16)
1 − sin x + 1 + sin x
=
cos x = 27 × 15 × 1× 11
2
= = 9 × 3 × 3 × 5 × 11
cos x
= 2 secan x (Dapat dibuktikan)
= 92 × 5 × 11
tan x + cotan y
b.
cotan x + tan y = 9 55 cm2
sin x
cos x
+
cos y
sin y
Jadi, luas ∆PQR = 9 55 cm2.
= cos x sin y
sin x
+ cos y

sin x sin y cos y cos x


cos x
× sin y
+ sin y
× cos x
= cos x cos y sin y sin x
sin x
× cos y
+ cos y
× sin x

60 Ulangan Akhir Semester


8. a. ∠ABC = 180° – 45° – 75° = 60° Selanjutnya, grafik fungsi g 1(x) = sin 3x
Dalam segitiga ABC berlaku: π
digeser sejauh 12 satuan ke kiri untuk men-
AC BC
= π
sin ∠ABC sin ∠CAB dapatkan grafik fungsi g2(x) = sin 3(x + 12 ).
20 BC π
⇔ = Grafik fungsi g2(x) = sin 3(x + 12 ) digeser ke
sin 60° sin 45°
20 BC bawah 2 satuan searah sumbu Y untuk
⇔ 1
3 = 1
2 π
2 2
mendapatkan grafik g(x) = 2 sin 3(x + 12 ) – 2.
1
20 × 2
2 π )
⇔ BC = 1
3(x
+ 12
3 sin
2 )= 3x
Y g 2(x sin
)=
g 1(x
20 2 3 1
⇔ BC = × −π
3 3 12
π π
X
0 2π 5π 19π 23π
20 4 3 4 12
⇔ BC =
3 12
6 cm –1
3
20 –2
Jadi, panjang BC = 6 cm.
3
–3 π
g(x) = 2 sin 3(x + )–2
b. Luas segitiga ABC (s, sd, s): 12

1 c. Menggambar sketsa grafik fungsi h(x) =


L∆ABC = 2 × AC × BC × sin ∠BCA π
3 – cos (x – 4 ).
1 20 6
= 2 × 20 × × sin 75° π
3 Sketsa grafik fungsi h(x) = 3 – cos (x – 4 )
200 6 dapat diperoleh dengan menggambar grafik
= × 0,96
3
fungsi h 1 (x) = cos x terlebih dahulu.
= 64 6 cm2 Selanjutnya, grafik fungsi h1(x) = cos x digeser
π
Jadi, luas segitiga ABC = 64 6 cm2. sejauh 4 satuan ke kanan untuk men-
9. a. Menggambar sketsa grafik fungsi f(x) = π
dapatkan grafik fungsi h2(x) = cos (x – 4 ).
π
tan (x – 6 ). Untuk mendapatkan grafik fungsi
π π
Sketsa grafik fungsi f(x) = tan (x – 6 ) dapat h3(x) = – cos (x – 4 ), setiap ordinat grafik
diperoleh dengan menggambar grafik fungsi π
fungsi h2(x) = cos (x – 4 ) dikalikan dengan
f 1(x) = tan x terlebih dahulu, kemudian
π –1 dan digambar grafiknya. Grafik fungsi
menggesernya sejauh 6 satuan ke kanan. π
h3(x) = – cos (x – 4 ) digeser ke atas 3 satuan
π
Y f1(x) = tan (x – )
6 searah sumbu Y untuk mendapatkan grafik
3 π
h(x) = 3 – cos (x – 4 ).
2
Y π
h(x) = 3 – cos (x – )
1 4 4

π 2π 3
X
0 π 7π 13π x
6 2 cos
–1 6 6 )=
h 1(x h2(x) = cos (x –
π
)
4
1
–2
−π X
0 π π 3π π 5π 3π 7π 2π
4 4 2 2 4
b. Menggambar sketsa grafik fungsi –1
h3 (x) =
4 4

– cos (x
π – π)
Sketsa grafik fungsi g(x) = sin 3(x + 12
)–2 4

dapat diperoleh dengan menggambar grafik


fungsi g 1 (x) = sin 3x terlebih dahulu.

Matematika Kelas X 61
10. Grafik fung si tersebut mempunyai 2 kemungkinan b. Sebagai fungsi sinus.
persamaan yaitu fungsi sinus atau fungsi kosinus. Misalkan persamaan grafik fungsi y =
a. Sebagai fungsi kosinus. a sin b(x + c) + d.
Misalkan persamaan grafik fungsi Nilai ymaks = 4 dan nilai ymin = –2.
y = a cos b(x + c) + d. 1
Nilai ymaks = 4 dan nilai ymin = –2. Amplitudo fungsi = 2 (4 – (–2)) = 3.
1 Nilai a = 3.
Amplitudo fungsi = 2 (4 – (–2)) = 3.
Grafik fungsi mempunyai tepat 1 puncak dan
Nilai a = 3. π 2π
Grafik fungsi mempunyai tepat 1 puncak dan 1 lembah pada interval – 3 sampai dengan 3 .
π 2π 2π π
1 lembah pada interval – 3 sampai dengan 3 . Periode fungsi = 3 – (– 3 ) = π.
2π π
Periode fungsi = 3 – (– 3 ) = π. 2π
Nilai b = π = 2.

Nilai b = π = 2. Persamaan fungsi sementara
Persamaan fungsi sementara y = 3 sin 2(x + c) + d.
y = 3 cos 2(x + c) + d. Titik potong sumbu Y yaitu (titik (0, 0)) pada
Titik potong sumbu Y (yaitu titik (0, 3)) pada fungsi y = 3 sin 2x mengalami pergeseran
fungsi y = 3 cos 2x mengalami pergeseran π
ke kiri sejauh 12 satuan searah sumbu X dan
π
ke kiri sejauh 3 satuan searah sumbu X dan ke atas 1 satuan searah sumbu Y sehingga
ke atas 1 satuan searah sumbu Y sehingga π
nilai c = 12 dan d = 1.
π
nilai c = 3 dan d = 2. π
Persamaan fungsinya y = 3 sin 2(x + 12 ) + 1.
Jadi, persamaan fungsinya Jadi, persamaan fungsinya
π π
y = 3 cos 2(x + 3 ) + 1. y = 3 cos 2(x + 3 ) + 1 atau
π
y = 3 sin 2(x + 12 ) + 1.

62 Ulangan Akhir Semester

Anda mungkin juga menyukai